CDM Cases Flashcards

1
Q

16yo L knee pain x2mo. pain at front of knee, below kneecap, worse with activity (running, walking up steps). no clicking, nl strength, tender over medial and lateral patella. no pain on joint lines.

DDx?

A

patellar instability
patellofemoral pain syndrome
synovial plica

How well did you know this?
1
Not at all
2
3
4
5
Perfectly
2
Q

16yo L knee pain x2mo. pain at front of knee, below kneecap, worse with activity (running, walking up steps). no clicking, nl strength, tender over medial and lateral patella. no pain on joint lines.
occasionally buckles, pops on standing,

signs/symptoms of underlying dx?

A

movie sign or theater sign
pain with stairs
pain with squatting

How well did you know this?
1
Not at all
2
3
4
5
Perfectly
3
Q

16yo L knee pain x2mo. pain at front of knee, below kneecap, worse with activity (running, walking up steps). no clicking, nl strength, tender over medial and lateral patella. no pain on joint lines.
occasionally buckles, pops on standing,

next steps in management?

A

activity modification

referral to physical therapy

How well did you know this?
1
Not at all
2
3
4
5
Perfectly
4
Q

medial knee pain ddx

A

medial compartment arthritis
MCL sprain
meniscus injury
pes anserinus pain syndrome

How well did you know this?
1
Not at all
2
3
4
5
Perfectly
5
Q

lateral knee pain ddx

A

lateral compartment arthritis
LCL sprain
meniscus injury
iliotibial band syndrome

How well did you know this?
1
Not at all
2
3
4
5
Perfectly
6
Q

focal anterior knee pain ddx

A

patellar or quadriceps tendinitis
prepatellar or infrapatellar bursitis
osgood-schlatter disease
synovial plica

How well did you know this?
1
Not at all
2
3
4
5
Perfectly
7
Q

vague anterior knee pain ddx

A
chondromalacia patellae
patellofemoral pain
osteoarthritis exacerbation
chronic patellar dislocation/subluxation
referred from hip
avascular necrosis
patellar stress fracture
How well did you know this?
1
Not at all
2
3
4
5
Perfectly
8
Q

posterior knee pain ddx

A

popliteal (baker’s) cyst
popliteal artery aneurysm or entrapment
hamstring strain
gastrocnemius strain

How well did you know this?
1
Not at all
2
3
4
5
Perfectly
9
Q

lachman/anterior drawer test

A

tibia pulled forward relative to femur

tests ACL

How well did you know this?
1
Not at all
2
3
4
5
Perfectly
10
Q

posterior drawer

A

tibia pushed backward relative to femur

PCL

How well did you know this?
1
Not at all
2
3
4
5
Perfectly
11
Q

mcmurray/apley, thessaly

A

meniscus compressed between femur and tibia with twisting motion
meniscus

How well did you know this?
1
Not at all
2
3
4
5
Perfectly
12
Q

ober’s test

A

the IT band put on stretch with the patient lying on their side
iliotibial band

How well did you know this?
1
Not at all
2
3
4
5
Perfectly
13
Q

patellar apprehension test

A
patella pulled to each side to test how far it will go and if the patient feels that it will dislocate
patellar laxity (medial patellofemoral ligament)
How well did you know this?
1
Not at all
2
3
4
5
Perfectly
14
Q

varus stress

A

pressure placed medially to laterally on knee

LCL

How well did you know this?
1
Not at all
2
3
4
5
Perfectly
15
Q

valgus stress

A

pressure placed laterally to medially on knee

MCL

How well did you know this?
1
Not at all
2
3
4
5
Perfectly
16
Q

common treatment for ACL tear

A

surgery for younger individuals

conservative care and eventual replacement for older individuals with arthritis

How well did you know this?
1
Not at all
2
3
4
5
Perfectly
17
Q

common treatment for PCL tera

A

conservative therapy or surgery

How well did you know this?
1
Not at all
2
3
4
5
Perfectly
18
Q

common treatment for MCL sprain

A

conservative (therapy)

How well did you know this?
1
Not at all
2
3
4
5
Perfectly
19
Q

common treatment for LCL sprain

A

conservative (therapy) unless complete tear, then surgery

How well did you know this?
1
Not at all
2
3
4
5
Perfectly
20
Q

common treatment for meniscus tear

A

conservative (therapy) if degenerative or not causeing mechanical symptoms (catching, popping, or locking of the knee)
surgery for mechanical symptoms

How well did you know this?
1
Not at all
2
3
4
5
Perfectly
21
Q

common treatment for ITB syndrome

A

conservative (therapy)

How well did you know this?
1
Not at all
2
3
4
5
Perfectly
22
Q

42yo M with palpable thyroid nodule and otherwise normal thyroid. no sx thyroid.

most important next steps?

A

neck ultrasound

thyroid function test

How well did you know this?
1
Not at all
2
3
4
5
Perfectly
23
Q

42yo M with palpable thyroid nodule and otherwise normal thyroid. no sx thyroid.
TSH wnl, U/S reveals 2cm nodule with internal microcalcifications. FNA showed psammoma bodies.

what’s the diagnosis?

A

papillary thyroid cancer

How well did you know this?
1
Not at all
2
3
4
5
Perfectly
24
Q

42yo M with palpable thyroid nodule and otherwise normal thyroid. no sx thyroid.
TSH wnl, U/S reveals 2cm nodule with internal microcalcifications. FNA showed psammoma bodies.
undergoes total thyroidectomy for papillary thyroid cancer. now ha perioral numbness and tingling.

what’s the laboratory abnormality?

A

hypocalcemia

hypothyroidism

How well did you know this?
1
Not at all
2
3
4
5
Perfectly
25
Q

45yo F with heat intolerance and weight loss. more anxious, exophthalmos. worsened over several months. elevated FT4 and low TSH. thyroid smooth and grossly enlarged.

most likely diagnosis?

A

Grave’s disease

How well did you know this?
1
Not at all
2
3
4
5
Perfectly
26
Q

45yo F with heat intolerance and weight loss. more anxious, exophthalmos. worsened over several months. elevated FT4 and low TSH. thyroid smooth and grossly enlarged.

what’s associated with Graves’ disease?

A

diffuse palpable goiter with audible bruit
exophthalmos
pretibial myxedema

How well did you know this?
1
Not at all
2
3
4
5
Perfectly
27
Q

45yo F with heat intolerance and weight loss. more anxious, exophthalmos. worsened over several months. elevated FT4 and low TSH. thyroid smooth and grossly enlarged.
fails medical therapy and undergoes total thyroidectomy. desats on extubation, vocal cords closed.

what anatomic structure was injured?

A

recurrent laryngeal nerve

How well did you know this?
1
Not at all
2
3
4
5
Perfectly
28
Q

45yo AA male for routine BP check. recent dx, started on HCTZ 12.5mg qd. truck driver, no exercise, fast food, smokes. elev Cr , urine alb/cr ratio, hgbA1c. BP remains high.

what’s the next best step in management?

A

add losartan 25mg once a day.

How well did you know this?
1
Not at all
2
3
4
5
Perfectly
29
Q

45yo AA male for routine BP check. recent dx, started on HCTZ 12.5mg qd. truck driver, no exercise, fast food, smokes. elev Cr , urine alb/cr ratio, hgbA1c. BP remains high.

next visit in 1mo, what lab test should be obtained given recent med change?

A

basic metabolic panel to check renal function

How well did you know this?
1
Not at all
2
3
4
5
Perfectly
30
Q

45yo AA male for routine BP check. recent dx, started on HCTZ 12.5mg qd. truck driver, no exercise, fast food, smokes. elev Cr , urine alb/cr ratio, hgbA1c. BP remains high.

what lifestyle changes should be recommended as part of your balanced healthcare plan?

A

quit smoking
DASH diet
increase physical activity
weight loss

How well did you know this?
1
Not at all
2
3
4
5
Perfectly
31
Q

6mo F, initially diff breastfeeding, switched to formula, thin wispy hair. diaper rash. dropping weight in growth chart.

What condition can be diagnosed based on current information?

A

failure to thrive

How well did you know this?
1
Not at all
2
3
4
5
Perfectly
32
Q

6mo F, initially diff breastfeeding, switched to formula, thin wispy hair. diaper rash. dropping weight in growth chart.
spits up food, vomits, arches back, nonwatery soft stools. mom neg HIV. no travel. UTD milestones.

what lab or imaging workup would you order?

A

basic metabolic panel
complete blood count
urinalysis

How well did you know this?
1
Not at all
2
3
4
5
Perfectly
33
Q

risk factors for FTT

A

medical conditions: prematurity, developmental delay, congenital abnormalities, low birth weight, poor oral hygiene, reflux
psychosocial: disordered feeding, family stressors, family history of partner abuse, poor parenting skills, poverty, restricted diet (religious or other)

How well did you know this?
1
Not at all
2
3
4
5
Perfectly
34
Q

commonly recommended testing in failure to thrive (in general)

A
CBC
BMP
VBG
serum lactate
ammonia
bilirubin
glucose
urinalysis
How well did you know this?
1
Not at all
2
3
4
5
Perfectly
35
Q

commonly recommended testing in failure to thrive (specialized based on history)

A
HIV
mycobacterium tuberculosis
cystic fibrosis (lipase)
milk protein allergy (trial of diet alteration)
intestinal malabsorption (celiac panel)
congenital anomaly (lung/heart/kidney/bladder)
pyloric stenosis (abdominal ultrasound)
neglect (skeletal survey)
hyperthyroidism (TSH)
How well did you know this?
1
Not at all
2
3
4
5
Perfectly
36
Q

6mo F, initially diff breastfeeding, switched to formula, thin wispy hair. diaper rash. dropping weight in growth chart.
spits up food, vomits, arches back, nonwatery soft stools. mom neg HIV. no travel. UTD milestones.
labs wnl

what is most likely diagnosis for FTT?

A

gastroesophageal reflux disease

How well did you know this?
1
Not at all
2
3
4
5
Perfectly
37
Q

signs of GERD in infants

A
frequent regurgitation or vomiting
prolonged feeding
refusal to feed
back arching
postprandial irritability
How well did you know this?
1
Not at all
2
3
4
5
Perfectly
38
Q

signs and symptoms of GERD in infants that require further evaluation

A

FTT
fever
persistent forceful or bilious vomiting
apnea
lethargy/seizures/neurodevelopmental delay
persistent diarrhea or constipation or GI bleeding
abdominal tenderness/distention or hepatosplenomegaly

How well did you know this?
1
Not at all
2
3
4
5
Perfectly
39
Q

6mo F, initially diff breastfeeding, switched to formula, thin wispy hair. diaper rash. dropping weight in growth chart.
spits up food, vomits, arches back, nonwatery soft stools. mom neg HIV. no travel. UTD milestones.
labs wnl

most appropriate treatment?

A

lifestyle changes

can start cimetidine

How well did you know this?
1
Not at all
2
3
4
5
Perfectly
40
Q

12yo F noticed midline mass on neck that becomes inflamed intermittently then resolves spontaneously. 2cm well-defined. mass moves up with tongue protrusion and with swallowing

what is most likely dx?

A

thyroglossal duct cyst

How well did you know this?
1
Not at all
2
3
4
5
Perfectly
41
Q

thyroglossal duct cyst

A

midline neck mass
generally above thyroid cartilage
moves with swallowing

How well did you know this?
1
Not at all
2
3
4
5
Perfectly
42
Q

branchial cleft cyst

A

lateral neck mass, almost always anterior to the sternocleidomastoid
does not move with swallowing
usually diagnosed in childhood

How well did you know this?
1
Not at all
2
3
4
5
Perfectly
43
Q

head and neck cancer

A
usually diagnosed in adulthood
risk factors (smoking, HPV)
solid, firm, immobile
can be in any distribution, usually associated with lymphadenopathy
often an underlying mass or lesion
How well did you know this?
1
Not at all
2
3
4
5
Perfectly
44
Q

thyroid mass

A

within the thyroid

generally near midline

How well did you know this?
1
Not at all
2
3
4
5
Perfectly
45
Q

12yo F noticed midline mass on neck that becomes inflamed intermittently then resolves spontaneously. 2cm well-defined. mass moves up with tongue protrusion and with swallowing

next best steps in management?

A

detailed history with review of systems
referral to ENT for monitoring and possible excision
thorough physical exam

How well did you know this?
1
Not at all
2
3
4
5
Perfectly
46
Q

12yo F noticed midline mass on neck that becomes inflamed intermittently then resolves spontaneously. 2cm well-defined. mass moves up with tongue protrusion and with swallowing

what anatomical structure must be removed to ensure resolution of the cyst?

A

middle third of the hyoid bone

How well did you know this?
1
Not at all
2
3
4
5
Perfectly
47
Q

47yo M hx T2DM. VSS. no hx anemia or CKD. baseline Cr 1.1.

what portions of ROS important for current health status given his diabetes?

A
cardiovascular
constitutional
eye
genitourinary
integumentary/skin
neurological
How well did you know this?
1
Not at all
2
3
4
5
Perfectly
48
Q

47yo M hx T2DM. VSS. no hx anemia or CKD. baseline Cr 1.1.
meds: Metformin 50mg BID, simvastatin 20mg, aspirin 81mg
3mo ago:HgbA1c 7.3; LDL 165; BP 146/84
today: Hgba1C 7.7; LDL 170; Cr 1.23; BP 145/95
ASCVD risk 12%
not compliant with diet/exercise routine. eating out more, work stress. has gym membership.

next best steps in management?

A

increase metformin
start lisinopril
change simvastatin to atorvastatin 40mg daily
recommend further lifestyle modifications

How well did you know this?
1
Not at all
2
3
4
5
Perfectly
49
Q

47yo M hx T2DM. VSS. no hx anemia or CKD. baseline Cr 1.1.
meds: Metformin 50mg BID, simvastatin 20mg, aspirin 81mg
3mo ago:HgbA1c 7.3; LDL 165; BP 146/84
today: Hgba1C 7.7; LDL 170; Cr 1.23; BP 145/95
ASCVD risk 12%
not compliant with diet/exercise routine. eating out more, work stress. has gym membership.
1mo later after starting lisinopril. dry cough.

most appropriate next step?

A

discontinue lisinopril and start an ARB

How well did you know this?
1
Not at all
2
3
4
5
Perfectly
50
Q

4yo F wcc. no pmhx. UTD vax. +daycare. rarely goes outside and plays and has never ridden tricycle. BMI 90%ile. vss.

what vaccines at 4yo visit?

A

Dtap (diphtheria, tetanus, pertussis)
MMR
polio (IPV)
varicella

How well did you know this?
1
Not at all
2
3
4
5
Perfectly
51
Q

4yo F wcc. no pmhx. UTD vax. +daycare. rarely goes outside and plays and has never ridden tricycle. BMI 90%ile. vss.

what milestones to ask for 4yo exam?

A

draw a person with 2-4 body parts
hopping and standing on one foot for up to 2 seconds
knows first and last name
sing a song from memory

How well did you know this?
1
Not at all
2
3
4
5
Perfectly
52
Q

age 1 milestones

A

cries when mom or dad leaves
repeats sounds or actions to get attention
says mama and dada and exclamations like “uh-oh”
tries to say words you say
gets to a sitting position without help
pulls up to stand, walks holding on to furniture (“cruising”)

How well did you know this?
1
Not at all
2
3
4
5
Perfectly
53
Q

age 2 milestones

A

copies others, especially adults and older children
shows defiant behavior
points to things or pictures when they are named
says sentences with 2-4 words
builds towers of 4 or more blocks
stands on tiptoe
begins to run

How well did you know this?
1
Not at all
2
3
4
5
Perfectly
54
Q

age 3 milestones

A

separates easily from mom and dad
says first name, age, sex
talks well enough for strangers to understand most of the time
carries on a conversation using 2-3 sentences
screws and unscrews jar lids or turns door handle
copies a circle with a pencil or crayon

How well did you know this?
1
Not at all
2
3
4
5
Perfectly
55
Q

age 4 milestones

A

should be able to recite their full name
like to play with other children
can sin ga song like ‘itsy bitsy spider’ by themselves
can draw a person with 2-4 body parts
can begin to hop and stand on 1 leg for short periods of time
can catch a bounced ball most of the time
can play board or card games

How well did you know this?
1
Not at all
2
3
4
5
Perfectly
56
Q

age5 milestones

A

wants to please and be like friends
can tell what’s real and what’s make believe
tells a simple story using full sentences
can draw a person with at least 6 body parts
counts 10 or more things
hops; may be able to skip
can use the toilet on their own
knows address

How well did you know this?
1
Not at all
2
3
4
5
Perfectly
57
Q

4yo F wcc. no pmhx. UTD vax. +daycare. rarely goes outside and plays and has never ridden tricycle. BMI 90%ile. vss.

what advice would you give the family regarding patient’s weight and physical activities?

A

she should do a variety of physical activities

she should limit sugary drinks

How well did you know this?
1
Not at all
2
3
4
5
Perfectly
58
Q

18yoF. well visit. starting college. UTD vax. sexually active since 14 w 4 male partners. all STI testing negative. always used condoms. no pmhx. OCPs.

when should she have her first pap smear?

A

21

How well did you know this?
1
Not at all
2
3
4
5
Perfectly
59
Q

18yoF. well visit. starting college. UTD vax. sexually active since 14 w 4 male partners. all STI testing negative. always used condoms. no pmhx. OCPs.
small lump in lower lateral quadrant. sometimes tender/painful that changes with menstrual cycle. possible nodularity.

what is the best initial test for this breast lump?

A

breast ultrasound

How well did you know this?
1
Not at all
2
3
4
5
Perfectly
60
Q

18yoF. well visit. starting college. UTD vax. sexually active since 14 w 4 male partners. all STI testing negative. always used condoms. no pmhx. OCPs.
small lump in lower lateral quadrant. sometimes tender/painful that changes with menstrual cycle. possible nodularity.
breast u/s: 2cm mass, without cystic features concerning for malignancy

what most important step in determining management of this condition?

A

core needle biopsy

How well did you know this?
1
Not at all
2
3
4
5
Perfectly
61
Q

3yo M. +SOB, runny nose for a few days. no change appetite. no diarrhea. +vomit. inc wet diapers and frequency. born @36wks, UTD vax, no sig pmhx. no smoke exposure. awake/alert. dry mucus membranes, mild abd tenderness, tachycardia

what tests for initial workup?

A

basic metabolic panel
complete blood count
urinalysis

How well did you know this?
1
Not at all
2
3
4
5
Perfectly
62
Q

3yo M. +SOB, runny nose for a few days. no change appetite. no diarrhea. +vomit. inc wet diapers and frequency. born @36wks, UTD vax, no sig pmhx. no smoke exposure. awake/alert. dry mucus membranes, mild abd tenderness, tachycardia
gluc 450. cxr neg; bicarb 8; K 5.0; u/a +ketones, +glucose

next best steps management?

A

admission to the hospital under pediatrics service
intravenous fluid bolus
intravenous insulin

How well did you know this?
1
Not at all
2
3
4
5
Perfectly
63
Q

76yo F new SOB for few weeks. dec exercise tolerance, takes more rest, inc LE swelling. pmhx HTN.
meds: lisinopril, HCTZ, amlodipine. misses doses.
BP 174/94; RR 18; O2 87% RA. tachy, 3/6 systolic murmur at apex radiates to axilla, irregular S1 w variable intensity. dec breath sounds b/l, scattered rales, mild pitting bilateral edema

what tests will you order?

A

electrocardiogram
chest x0ray
brain natriuretic peptide (BNP)

How well did you know this?
1
Not at all
2
3
4
5
Perfectly
64
Q

76yo F new SOB for few weeks. dec exercise tolerance, takes more rest, inc LE swelling. pmhx HTN.
meds: lisinopril, HCTZ, amlodipine. misses doses.
BP 174/94; RR 18; O2 87% RA. tachy, 3/6 systolic murmur at apex radiates to axilla, irregular S1 w variable intensity. dec breath sounds b/l, scattered rales, mild pitting bilateral edema
BNP 5900. troponin T 39. ekg +afib wRVR.

what additional testing?

A

echocardiogram

thyroid stimulating hormone level

How well did you know this?
1
Not at all
2
3
4
5
Perfectly
65
Q

workup for atrial fibrillation

A
cbc
electrolytes
renal function
TSH
troponin (if ischemia suspected)
echocardiogram
How well did you know this?
1
Not at all
2
3
4
5
Perfectly
66
Q

possible causes of atrial fibrillation

A
heart disease (hypertrophic, ischemic, hypertensive, valvular, congenital)
thromboembolic disease
obstructive sleep apnea
obesity
diabetes
CKD
hypo- or hyperthyroidism
infection
electrolyte disorders
drug use
surgical stress (often cardiac surgery)
How well did you know this?
1
Not at all
2
3
4
5
Perfectly
67
Q

76yo F new SOB for few weeks. dec exercise tolerance, takes more rest, inc LE swelling. pmhx HTN.
meds: lisinopril, HCTZ, amlodipine. misses doses.
BP 174/94; RR 18; O2 87% RA. tachy, 3/6 systolic murmur at apex radiates to axilla, irregular S1 w variable intensity. dec breath sounds b/l, scattered rales, mild pitting bilateral edema
BNP 5900. troponin T 39. ekg +afib wRVR.
echo: LVH and nl LV systolic fxn. LA enlargement, mod MR, no effusion. TSH, HgbA1c, and lipid obtained

most appropriate next steps in management of patient?

A

diltiazem, PO
furosemide IV
heparin nomogram, IV

How well did you know this?
1
Not at all
2
3
4
5
Perfectly
68
Q

management of afib w rapid ventricular rate if unstable (ischemia, hypotension, or severe heart failure)

A

cardoversion

How well did you know this?
1
Not at all
2
3
4
5
Perfectly
69
Q

management of afib w rapid ventricular rate if stable

A

rate control with ccb or bb
initiate anticoagulation
if symptoms < 48hrs or TEE no apical thrombus => consider cardioversion
if symptoms >48hrs or no TEE or TEE no apical thrombus => anticoagulate 3+ weeks prior to cardioversion

How well did you know this?
1
Not at all
2
3
4
5
Perfectly
70
Q

76yo F new SOB for few weeks. dec exercise tolerance, takes more rest, inc LE swelling. pmhx HTN.
meds: lisinopril, HCTZ, amlodipine. misses doses.
BP 174/94; RR 18; O2 87% RA. tachy, 3/6 systolic murmur at apex radiates to axilla, irregular S1 w variable intensity. dec breath sounds b/l, scattered rales, mild pitting bilateral edema
BNP 5900. troponin T 39. ekg +afib wRVR.
echo: LVH and nl LV systolic fxn. LA enlargement, mod MR, no effusion. TSH, HgbA1c, and lipid obtained
started on apixiban. HgbA1c 5.6; other labs wnl. continued bp issues. d/c on chlorthalidone and amlodipine.

based on information currently available, what is the CHA2DS2-VASc score for the patient?

A
5
age(2)
heart failure (1)
HTN (1)
female (1)
How well did you know this?
1
Not at all
2
3
4
5
Perfectly
71
Q

CHA2DS2-VASc score

A

(C) CHF history = +1
(H) hypertension = +1
(A2) age (A2): <65 = 0; 65-74 = +1; >75 = +2
(D) diabetes = +1
(S2) stroke/TIA/thromboembolism (S2): +2
(VA) vascular disease (PAD, aortic plaque, prior MI) +1
(Sc) sex category (female) +1

How well did you know this?
1
Not at all
2
3
4
5
Perfectly
72
Q

CHA2DS2-VASc score vs. stroke risk

A
0 -> 0%
1 -> 1.3%
2 -> 2.2%
3 -> 3.2%
4 -> 4.0%
5 -> 6.7%
6 -> 9.8%
7 -> 9.6%
8 -> 12.5%
9 -> 15.2%
How well did you know this?
1
Not at all
2
3
4
5
Perfectly
73
Q

31yo F ED new onset confusion. nonsensical statements, hallucinating. pmhx diabetes. uses multiple substances incl cocaine, heroin. frequent alcohol user (up to 2 pints per day). 3d camping with natural cleanse. yesterday sweaty and tremulous. exam: sweaty, tremulous, talking to wall, on tangetns, no abd pain, glucose 187.

most important initial steps in workup?

A

liver function tests
basic metabolic panel
complete blood count

How well did you know this?
1
Not at all
2
3
4
5
Perfectly
74
Q

31yo F ED new onset confusion. nonsensical statements, hallucinating. pmhx diabetes. uses multiple substances incl cocaine, heroin. frequent alcohol user (up to 2 pints per day). 3d camping with natural cleanse. yesterday sweaty and tremulous. exam: sweaty, tremulous, talking to wall, on tangetns, no abd pain, glucose 187.
ekg sinus tachy w QT 457; betahydroxybutyrate elev, LFTs minimally elevated, UDS neg. repeat exam agitated, tachy, tremulous, diaphoretic

next best steps in management?

A

IV lorazepam
admission to the hospital
thiamine

How well did you know this?
1
Not at all
2
3
4
5
Perfectly
75
Q

66yo M in mid Dec. not seen since early 50s. +smoker, +alcohol. healthy diet, no exercise. +omeprazole.
denies vax since last visit.

what vaccines are most appropriate to give at today’s visit?

A

influenza
PPSV23
Tdap
zoster

How well did you know this?
1
Not at all
2
3
4
5
Perfectly
76
Q

66yo M in mid Dec. not seen since early 50s. +smoker, +alcohol. healthy diet, no exercise. +omeprazole.
denies vax since last visit.

what screening tests?

A

abdominal ultrasound
colonoscopy
CT of thorax, low dose

How well did you know this?
1
Not at all
2
3
4
5
Perfectly
77
Q

66yo M in mid Dec. not seen since early 50s. +smoker, +alcohol. healthy diet, no exercise. +omeprazole.
denies vax since last visit.
+metabolic syndrome.

what are components of metabolic syndrome according to Adult Treatment Panel III?

A

serum TGs >150 or drug tx for elevated TGs
serum HDL <40 in men and <50 in women or drug tx for low HDL
fasting plasma glucose >100 or drug tx for elevated blood glucose

abd obesity >40in men, >35in women
BP >130/85 or drug tx for HTN

How well did you know this?
1
Not at all
2
3
4
5
Perfectly
78
Q

67yoM x2d constipation, LLQ pain. 8/10, continuous, nonradiating. no BM, but +flatus. no hx surg. no hematochezia or melena. famhx +colon cancer. -smoking. T 101.5. TTP LLQ w/o rebound. mild vol guarding.

most likely diagnosis?

A

acute diverticulitis

How well did you know this?
1
Not at all
2
3
4
5
Perfectly
79
Q

67yoM x2d constipation, LLQ pain. 8/10, continuous, nonradiating. no BM, but +flatus. no hx surg. no hematochezia or melena. famhx +colon cancer. -smoking. T 101.5. TTP LLQ w/o rebound. mild vol guarding.
dx diverticulitis. start IVF. elev wbc, nl Cr. electrolytes stable

what further workup should be performed?

A

CT abdomen pelvis with IV and oral contrast

How well did you know this?
1
Not at all
2
3
4
5
Perfectly
80
Q

67yoM x2d constipation, LLQ pain. 8/10, continuous, nonradiating. no BM, but +flatus. no hx surg. no hematochezia or melena. famhx +colon cancer. -smoking. T 101.5. TTP LLQ w/o rebound. mild vol guarding.
dx diverticulitis. start IVF. elev wbc, nl Cr. electrolytes stable
Hgb 10. CT -> thickening sigmoid colon w/o perf or abscess. T 102.4. BP 94/62. HR 118. s/p 1L NS, antiemetics, IV morphine

what are the most appropriate next steps in management?

A

IV ceftriaxone and metronidazole
IV fluid resuscitation
patient should be made nil per os (NPO)

How well did you know this?
1
Not at all
2
3
4
5
Perfectly
81
Q

67yoM x2d constipation, LLQ pain. 8/10, continuous, nonradiating. no BM, but +flatus. no hx surg. no hematochezia or melena. famhx +colon cancer. -smoking. T 101.5. TTP LLQ w/o rebound. mild vol guarding.
dx diverticulitis. start IVF. elev wbc, nl Cr. electrolytes stable
Hgb 10. CT -> thickening sigmoid colon w/o perf or abscess. T 102.4. BP 94/62. HR 118. s/p 1L NS, antiemetics, IV morphine
resolves. f/u PCP in 2wks feeling at baseline. asks why he developed diverticulitis and what he can do moving forward.

what further therapy, if any, should be considered as part of his follow-up care?

A

colonoscopy

How well did you know this?
1
Not at all
2
3
4
5
Perfectly
82
Q

67yo F. pmhx HTN, afib, HLD. acute hematemesis +mild confusion, abd pain x2d. started 2hrs ago. x3 episodes nonbloody, became bloody and had 4 large volume. ate salmon at a restaurant and initially thought food poisoning. +diarrhea, +cramping. +dark stools. +drink 5th/day x7yrs. -smoking. +lightheaded/dizzy
T 99.1; BP 102/40. HR 113. RR 22. pale, older. jaundice, distended abd, pitting LE edema. +asterixis

what are some of the questions from the HPI and PMHx that would be important to know to allow for optimal treatment of her acute bleed?

A

Required:

  • is the patient currently taking any anticoagulation?
  • has the patient had a prior EGD, GI bleed, or hx PUD or varices?

Correct responses:

  • does the patient have a known history of cirrhosis?
  • is the patient on any anticoagulation?
  • does the patient have any NSAID use?
  • has the patient had a previous endoscopy?
  • is she using antiplatelet agents such as aspirin?
  • does the patient have any known bleeding disorders?
  • does the patient have odynophagia?
  • previous history of H pylori?
How well did you know this?
1
Not at all
2
3
4
5
Perfectly
83
Q

67yo F. pmhx HTN, afib, HLD. acute hematemesis +mild confusion, abd pain x2d. started 2hrs ago. x3 episodes nonbloody, became bloody and had 4 large volume. ate salmon at a restaurant and initially thought food poisoning. +diarrhea, +cramping. +dark stools. +drink 5th/day x7yrs. -smoking. +lightheaded/dizzy
T 99.1; BP 102/40. HR 113. RR 22. pale, older. jaundice, distended abd, pitting LE edema. +asterixis
too encephalopathic. no doc for a few yrs. no hx endoscopy. int melena x1wk. dx last year with “liver failure.” daily ibuprofen for headaches since younger.

what are the most important etiologies of bleeding that need to be considered for treatment?

A

peptic ulcer disease

esophageal/hepatic varices

How well did you know this?
1
Not at all
2
3
4
5
Perfectly
84
Q

67yo F. pmhx HTN, afib, HLD. acute hematemesis +mild confusion, abd pain x2d. started 2hrs ago. x3 episodes nonbloody, became bloody and had 4 large volume. ate salmon at a restaurant and initially thought food poisoning. +diarrhea, +cramping. +dark stools. +drink 5th/day x7yrs. -smoking. +lightheaded/dizzy
T 99.1; BP 102/40. HR 113. RR 22. pale, older. jaundice, distended abd, pitting LE edema. +asterixis
too encephalopathic. no doc for a few yrs. no hx endoscopy. int melena x1wk. dx last year with “liver failure.” daily ibuprofen for headaches since younger.
RUQ u/s 5yrs ago w nodular liver. pt obtunded. T 101.8. BP 78/36. HR 122. RR 24.

besides variceal bleeding, what are the complications of cirrhosis that we must consider either evaluating for now or which may impact our immediate treatment of this patient going forward?

A

ascites
hepatic encephalopathy
spontaneous bacterial peritonitis

How well did you know this?
1
Not at all
2
3
4
5
Perfectly
85
Q

67yo F. pmhx HTN, afib, HLD. acute hematemesis +mild confusion, abd pain x2d. started 2hrs ago. x3 episodes nonbloody, became bloody and had 4 large volume. ate salmon at a restaurant and initially thought food poisoning. +diarrhea, +cramping. +dark stools. +drink 5th/day x7yrs. -smoking. +lightheaded/dizzy
T 99.1; BP 102/40. HR 113. RR 22. pale, older. jaundice, distended abd, pitting LE edema. +asterixis
too encephalopathic. no doc for a few yrs. no hx endoscopy. int melena x1wk. dx last year with “liver failure.” daily ibuprofen for headaches since younger.
RUQ u/s 5yrs ago w nodular liver. pt obtunded. T 101.8. BP 78/36. HR 122. RR 24.
Hgb 5.7. Plt 78. INR 1.4. has not filled meds in 6mo. no anticoagulant.

what are the next most important steps in management?

A

Required:

  • blood transfusion
  • IV fluid resuscitation
  • IV proton pump inhibitors
  • IV octreotide
  • IV ceftriaxone (3rd gen ceph or FQ or pip/taz)
  • consult gastroenterology for EGD

correct:

  • 2 large-bore peripheral IVs
  • IV fluid resuscitation with crystalloid
  • blood transfusion
  • complete blood count
  • proton pump inhibitor infusion
  • IV octreotide
  • ICU admission
  • IV ceftriaxone
  • plan for EGD
  • diagnostic pericentesis
How well did you know this?
1
Not at all
2
3
4
5
Perfectly
86
Q

32yo F w rash on face, shoulders, arms x3mo. wrose when outside. occasional mouth sores and joint pain x2yrs mostly in fingers/ankles. famhx arthritis, “eczema.” mom hosp B cell lymphoma and father died of MI at 47. -alcohol. -smoke. comfortable. rash is erythematous, annular, and exhibits scaling. localized to cheeks, neck, and lower arms. hypopigmentation. joints swollen with synovitis and mild erythema. small mucositis with superficial ulceration

what is the patient’s most likely underlying diagnosis?

A

systemic lupus erythematosus (SLE)

How well did you know this?
1
Not at all
2
3
4
5
Perfectly
87
Q

32yo F w rash on face, shoulders, arms x3mo. wrose when outside. occasional mouth sores and joint pain x2yrs mostly in fingers/ankles. famhx arthritis, “eczema.” mom hosp B cell lymphoma and father died of MI at 47. -alcohol. -smoke. comfortable. rash is erythematous, annular, and exhibits scaling. localized to cheeks, neck, and lower arms. hypopigmentation. joints swollen with synovitis and mild erythema. small mucositis with superficial ulceration

which of the following tests should be obtained in this patient as part of the initial workup?

A

antinuclear antibody
basic metabolic panel
complete blood count

How well did you know this?
1
Not at all
2
3
4
5
Perfectly
88
Q

32yo F w rash on face, shoulders, arms x3mo. wrose when outside. occasional mouth sores and joint pain x2yrs mostly in fingers/ankles. famhx arthritis, “eczema.” mom hosp B cell lymphoma and father died of MI at 47. -alcohol. -smoke. comfortable. rash is erythematous, annular, and exhibits scaling. localized to cheeks, neck, and lower arms. hypopigmentation. joints swollen with synovitis and mild erythema. small mucositis with superficial ulceration
wbc 2.5. hgb 11.8. plt 98. Cr 0.5. ESR 88.

what medication is considered first line for longterm control of this patient’s underlying condition?

A

oral hydroxychloroquine

How well did you know this?
1
Not at all
2
3
4
5
Perfectly
89
Q

32yo F w rash on face, shoulders, arms x3mo. wrose when outside. occasional mouth sores and joint pain x2yrs mostly in fingers/ankles. famhx arthritis, “eczema.” mom hosp B cell lymphoma and father died of MI at 47. -alcohol. -smoke. comfortable. rash is erythematous, annular, and exhibits scaling. localized to cheeks, neck, and lower arms. hypopigmentation. joints swollen with synovitis and mild erythema. small mucositis with superficial ulceration
wbc 2.5. hgb 11.8. plt 98. Cr 0.5. ESR 88.
started on HCQ. 3wks later +SOB, chest pain, fever. intubated. cxr b/l fluffy infiltrates. infectious w/u neg. echo w preserved EF. coarse rhonchi and rales, scattered wheezing. adm labs wbc 1.74. hgb 7.2. plt 55k. PT 12. Cr 1.67. uRBCs 60. C4 6.

what physiologic process is most consistent with the patient’s underlying diagnosis?

A

high diffusing capacity for carbon monoxide

catastrophic pulmonary complication of lupus is diffuse alveolar hemorrhage

How well did you know this?
1
Not at all
2
3
4
5
Perfectly
90
Q

32yo F w rash on face, shoulders, arms x3mo. wrose when outside. occasional mouth sores and joint pain x2yrs mostly in fingers/ankles. famhx arthritis, “eczema.” mom hosp B cell lymphoma and father died of MI at 47. -alcohol. -smoke. comfortable. rash is erythematous, annular, and exhibits scaling. localized to cheeks, neck, and lower arms. hypopigmentation. joints swollen with synovitis and mild erythema. small mucositis with superficial ulceration
wbc 2.5. hgb 11.8. plt 98. Cr 0.5. ESR 88.
started on HCQ. 3wks later +SOB, chest pain, fever. intubated. cxr b/l fluffy infiltrates. infectious w/u neg. echo w preserved EF. coarse rhonchi and rales, scattered wheezing. adm labs wbc 1.74. hgb 7.2. plt 55k. PT 12. Cr 1.67. uRBCs 60. C4 6.
3d high dose methylprednisolone and cyclophosphamide. 8th day of hosp, develops R LE pain, white toes that turn blue. vasc surg consulted and undergoes CT angio -> acute arterial occlusion of peroneal and comon femoral artery of RLE. urgent endarterectoy. susp underlying atherosclorosis

what hematologic abnormality is associated with the cause of this patient’s arterial clot?

A

increased partial thromboplastin time

How well did you know this?
1
Not at all
2
3
4
5
Perfectly
91
Q

systemic complications of SLE: constitutional

A

fever, fatigue, myalgia, weight loss

How well did you know this?
1
Not at all
2
3
4
5
Perfectly
92
Q

systemic complications of SLE: musculoskeletal

A

arthritis, often migratory, polyarticular, and symmetrical

How well did you know this?
1
Not at all
2
3
4
5
Perfectly
93
Q

systemic complications of SLE: skin

A

malar rash, oral ulcers, photosensitivity

How well did you know this?
1
Not at all
2
3
4
5
Perfectly
94
Q

systemic complications of SLE: cardiac

A

raynaud phenomenon, vasculitis, pericarditis, Libman sacks endocarditis

How well did you know this?
1
Not at all
2
3
4
5
Perfectly
95
Q

systemic complications of SLE: hematologic

A

pancytopenia, thromboembolic disease (with or without antiphospholipid syndrome)

How well did you know this?
1
Not at all
2
3
4
5
Perfectly
96
Q

systemic complications of SLE: neurologic

A

CNS lupus or lupus cerebritis

How well did you know this?
1
Not at all
2
3
4
5
Perfectly
97
Q

systemic complications of SLE: ophthalmologic

A

keratoconjunctivitis sicca (secondary to Sjogren’s syndrome), but any eye structure can be inflamed or involved

How well did you know this?
1
Not at all
2
3
4
5
Perfectly
98
Q

systemic complications of SLE: pulmonary

A

diffuse alveolar hemorrhage, interstitial lung disease

How well did you know this?
1
Not at all
2
3
4
5
Perfectly
99
Q

systemic complications of SLE: renal

A

nephritis

How well did you know this?
1
Not at all
2
3
4
5
Perfectly
100
Q

32yo F w rash on face, shoulders, arms x3mo. wrose when outside. occasional mouth sores and joint pain x2yrs mostly in fingers/ankles. famhx arthritis, “eczema.” mom hosp B cell lymphoma and father died of MI at 47. -alcohol. -smoke. comfortable. rash is erythematous, annular, and exhibits scaling. localized to cheeks, neck, and lower arms. hypopigmentation. joints swollen with synovitis and mild erythema. small mucositis with superficial ulceration
wbc 2.5. hgb 11.8. plt 98. Cr 0.5. ESR 88.
started on HCQ. 3wks later +SOB, chest pain, fever. intubated. cxr b/l fluffy infiltrates. infectious w/u neg. echo w preserved EF. coarse rhonchi and rales, scattered wheezing. adm labs wbc 1.74. hgb 7.2. plt 55k. PT 12. Cr 1.67. uRBCs 60. C4 6.
3d high dose methylprednisolone and cyclophosphamide. 8th day of hosp, develops R LE pain, white toes that turn blue. vasc surg consulted and undergoes CT angio -> acute arterial occlusion of peroneal and comon femoral artery of RLE. urgent endarterectoy. susp underlying atherosclorosis
anti-beta-2-glycoprotein antibody IgG >180. dilute russell viper venom time 88sec (elev)

given dx of antiphospholipid syndrome, which anticoagulant is most appropriate for long term treatment of this condition?

A

oral warfarin

How well did you know this?
1
Not at all
2
3
4
5
Perfectly
101
Q

exceptions to the use of direct oral anticoagulants

A
antiphospholipid syndrome
mechanical heart valves
atrial fibrillation due to mitral stenosis
severe renal or liver disease
pregnancy
cancer
How well did you know this?
1
Not at all
2
3
4
5
Perfectly
102
Q

tx antiphosphlipid syndrome

A

warfarin or unfractionated heparin or LMWH

How well did you know this?
1
Not at all
2
3
4
5
Perfectly
103
Q

tx mechanical heart valves

A

warfarin or UFH

How well did you know this?
1
Not at all
2
3
4
5
Perfectly
104
Q

tx a fib d/t mitral stenosis

A

warfarin or UFH or LMWH

How well did you know this?
1
Not at all
2
3
4
5
Perfectly
105
Q

tx severe renal or liver disease

A

consider warfarin
consider UFH
LMWH c/i renal failure

How well did you know this?
1
Not at all
2
3
4
5
Perfectly
106
Q

tx pregnancy

A

LMWH or UFH

How well did you know this?
1
Not at all
2
3
4
5
Perfectly
107
Q

tx cancer

A

DOACs have been recently shown to be equivalent to LMWH, though data is new, so choose LMWH

108
Q

19yo F. suprapubic pain x2d. intermittent 4/10. +dysuria. +frequency. -sex. -McBurney’s. -rebound.

which of the following tests would you order as part of initial workup?

A

urinalysis

109
Q

19yo F. suprapubic pain x2d. intermittent 4/10. +dysuria. +frequency. -sex. -McBurney’s. -rebound.
u/a 2+ nitrite, +leuks, -RBCs. clx pending

which of the following treatments is appropriate for this patient at this time?

A

nitrofurantoin

other options: TMP/SMX, cefalexin, ciprofloxacin, fosfomycin

110
Q

19yo F. suprapubic pain x2d. intermittent 4/10. +dysuria. +frequency. -sex. -McBurney’s. -rebound.
u/a 2+ nitrite, +leuks, -RBCs. clx pending
3x over 1yr for recurrent UTIs. pan-sens E. coli. worsening UTI sx. more fatigued. +n/v. +flank pain. T 102.3. R CVA tender.

what is the most likely diagnosis?

A

pyelonephritis

111
Q

19yo F. suprapubic pain x2d. intermittent 4/10. +dysuria. +frequency. -sex. -McBurney’s. -rebound.
u/a 2+ nitrite, +leuks, -RBCs. clx pending
3x over 1yr for recurrent UTIs. pan-sens E. coli. worsening UTI sx. more fatigued. +n/v. +flank pain. T 102.3. R CVA tender.
in ED: Cr 0.89. mild hypoNa, hypoCl w metabolic acidosis. elev lactate, u/a +leuks and +nitrites, 3+ blood w 32 RBCs.

what other workup should be obtained?

A

CT abdomen and pelvis with IV contrast

- test of choice to detect infectious or mechanical complications assoc with complicated UTIs

112
Q

19yo F. suprapubic pain x2d. intermittent 4/10. +dysuria. +frequency. -sex. -McBurney’s. -rebound.
u/a 2+ nitrite, +leuks, -RBCs. clx pending
3x over 1yr for recurrent UTIs. pan-sens E. coli. worsening UTI sx. more fatigued. +n/v. +flank pain. T 102.3. R CVA tender.
in ED: Cr 0.89. mild hypoNa, hypoCl w metabolic acidosis. elev lactate, u/a +leuks and +nitrites, 3+ blood w 32 RBCs.
CT abd/pelvis: b/l renal hypodensities and fat stranding consistent with pyelo. sig R hydronephrosis with 0.9cm renal stone at R ureteropelvic junction

what are the next steps in management of this patient?

A

consult to urology for urgent evaluation for stenting and stone removal
IV ceftriaxone

113
Q

13yo F w back pain x2wks. elite gymnast. mostly midline, may be R of center. radiating to R buttocks and back of thigh. Worse with extension. exacerbated when standing on R leg. L4/L5.

which of the following conditions would be highest on your differential diagnosis?

A

apophysitis of the vertebral body

spondylolysis

114
Q

13yo F w back pain x2wks. elite gymnast. mostly midline, may be R of center. radiating to R buttocks and back of thigh. Worse with extension. exacerbated when standing on R leg. L4/L5.

which of the following would be the appropriate firstline imaging study for the suspected diagnosis?

A

plain radiographs of the lumbar spine

115
Q

13yo F w back pain x2wks. elite gymnast. mostly midline, may be R of center. radiating to R buttocks and back of thigh. Worse with extension. exacerbated when standing on R leg. L4/L5.
dx with spondylolysis, no evidence of spondylolisthesis.

what treatment options should you consider?

A

acetaminophen
physical therapy
rest from gymnastics

116
Q

55yo M in ED w lightheaded, +n/v. 3 episodes of vomiting dark maroon/blood. stool black and sticky. pmhx chronic LBP relieved w PT and heating pads. recently taking naproxen 4-5x/day. BP 92/58, HR 112. PE: pale, melena on rectal exam

Which tests will you order for this patient at this time?

A

basic metabolic panel
complete blood count
type and screen

117
Q

55yo M in ED w lightheaded, +n/v. 3 episodes of vomiting dark maroon/blood. stool black and sticky. pmhx chronic LBP relieved w PT and heating pads. recently taking naproxen 4-5x/day. BP 92/58, HR 112. PE: pale, melena on rectal exam
Hgb 6; Hct 28%; K 3.4; Cr 1.4; BUN 76; Gluc 106; sinus tach, INR 1.2

next most important steps in management?

A

blood transfusion
start IV omeprazole
consult gastroenterology for EGD

118
Q

upper GI bleed labs

A
CBC
BMP
Type and screen
LFTs
troponin
119
Q

upper GI bleed other workup

A

EKG
chest or abdominal imaging (if concerned for perforation)
CT angio if embolization if needed

120
Q

upper GI bleed inpatient treatment

A
IV proton pump inhibitors (omeprazole)
IV octreotide (if concerned for varices)
antibiotics (if cirrhosis)
blood transfusions as indicated
fluid resuscitation
endoscopy (if no perforation)
121
Q

55yo M in ED w lightheaded, +n/v. 3 episodes of vomiting dark maroon/blood. stool black and sticky. pmhx chronic LBP relieved w PT and heating pads. recently taking naproxen 4-5x/day. BP 92/58, HR 112. PE: pale, melena on rectal exam
Hgb 6; Hct 28%; K 3.4; Cr 1.4; BUN 76; Gluc 106; sinus tach, INR 1.2
after resusc, upper GI -> bleeding ulcer in duodenum and scattered gastritis, -H. pylori. no further signs of bleeding.

what med changes should be made for outpatient treatment?

A

discontinue NSAIDs

start oral proton pump inhibitors

122
Q

upper GI bleed outpatient treatment

A

oral PPI for 4-8 weeks
stop all NSAIDs
stop alcohol/tobacco if related
treat H. pylori if positive
consider repeat EGD as needed to ensure healing
follow up biopsy results if concern for malignancy

123
Q

73yo M for annual. breathing more heavily past few weeks, needing more rest, no longer walks d/t fatigue. gained weight. sleeps on reclincer. hx HTN (poor control d/t med adherence) nosurg. drinks 3 beers nightly.

what other signs and symptoms would be considered “major criteria” when making a clinical diagnosis in this case?

A

distended neck veins
rales
waking up at night with acute shortness of breath
S3

124
Q

major criteria for heart failure

A

2+ of:

  • acute pulmonary edema
  • cardiomegaly
  • hepatojugular reflux
  • neck vein distention
  • paroxysmal nocturnal dyspnea or orthopnea
  • pulmonary rales
  • third heart sound (S3 gallop)
  • weight loss of >4.5kg in 5 days in response to treatment
125
Q

minor criteria for heart failure

A

1 major + 2 minor:

  • ankle edema
  • dyspnea on exertion
  • hepatomegaly
  • nocturnal cough
  • pleural effusion
  • tachycardia (>120)
126
Q

73yo M for annual. breathing more heavily past few weeks, needing more rest, no longer walks d/t fatigue. gained weight. sleeps on reclincer. hx HTN (poor control d/t med adherence) nosurg. drinks 3 beers nightly.
SOB moving from chair to exam table. c/o doing dishes and ADLs in morning. abd distention. S3 gallop, blowing holosystolic murmur with radiation to axilla. JVD, +hepatojugular reflux, +abd ascites. +b/l pitting edema to knee.

which combo of imaging modalities is best initial diagnostic step for this patient?

A

chest radiograph and transthoracic echocardiogram

127
Q

73yo M for annual. breathing more heavily past few weeks, needing more rest, no longer walks d/t fatigue. gained weight. sleeps on reclincer. hx HTN (poor control d/t med adherence) nosurg. drinks 3 beers nightly.
SOB moving from chair to exam table. c/o doing dishes and ADLs in morning. abd distention. S3 gallop, blowing holosystolic murmur with radiation to axilla. JVD, +hepatojugular reflux, +abd ascites. +b/l pitting edema to knee.
EKG NSR w LVH, no ischemic changes; Echo rEF 25%. cxr normal card size with small b/l pleural effusions and vasc congestion w/o covert pulmonary edema.

based on NYHA, how would you classify this patient?

A

class III

128
Q

NYHA functional classes

A

I: comfortable at rest, no sx activity, no limitations
II: comfortable at rest, sx with more than normal activity (mowing,vacuuming, brisk walking), slight limitations
III: comfortable at rest, sx with ordinary activites (ADLs), marked limitations in ADLs
IV: short of breath at rest, sx at rest, cannot do any activity without significant symptoms

129
Q

73yo M for annual. breathing more heavily past few weeks, needing more rest, no longer walks d/t fatigue. gained weight. sleeps on reclincer. hx HTN (poor control d/t med adherence) nosurg. drinks 3 beers nightly.
SOB moving from chair to exam table. c/o doing dishes and ADLs in morning. abd distention. S3 gallop, blowing holosystolic murmur with radiation to axilla. JVD, +hepatojugular reflux, +abd ascites. +b/l pitting edema to knee.
EKG NSR w LVH, no ischemic changes; Echo rEF 25%. cxr normal card size with small b/l pleural effusions and vasc congestion w/o covert pulmonary edema.

most specific diagnosis given currently available information?

A

heart failure with reduced ejection fraction

130
Q

73yo M for annual. breathing more heavily past few weeks, needing more rest, no longer walks d/t fatigue. gained weight. sleeps on reclincer. hx HTN (poor control d/t med adherence) nosurg. drinks 3 beers nightly.
SOB moving from chair to exam table. c/o doing dishes and ADLs in morning. abd distention. S3 gallop, blowing holosystolic murmur with radiation to axilla. JVD, +hepatojugular reflux, +abd ascites. +b/l pitting edema to knee.
EKG NSR w LVH, no ischemic changes; Echo rEF 25%. cxr normal card size with small b/l pleural effusions and vasc congestion w/o covert pulmonary edema.
undergoes cardiac cath w/ minimal CAD and markedly inc left sided pressures. +diuresis. nonischemic cardiomyopathy. referred to advanced heart failure clinic.

at his first appointment, they discuss lifestyle modifications. what lifestyle modification should be recommended to this patient?

A

alcohol cessation
moderate salt restriction
regular physical activity

131
Q

73yo M for annual. breathing more heavily past few weeks, needing more rest, no longer walks d/t fatigue. gained weight. sleeps on reclincer. hx HTN (poor control d/t med adherence) nosurg. drinks 3 beers nightly.
SOB moving from chair to exam table. c/o doing dishes and ADLs in morning. abd distention. S3 gallop, blowing holosystolic murmur with radiation to axilla. JVD, +hepatojugular reflux, +abd ascites. +b/l pitting edema to knee.
EKG NSR w LVH, no ischemic changes; Echo rEF 25%. cxr normal card size with small b/l pleural effusions and vasc congestion w/o covert pulmonary edema.
undergoes cardiac cath w/ minimal CAD and markedly inc left sided pressures. +diuresis. nonischemic cardiomyopathy. referred to advanced heart failure clinic.

which drugs improve mortality in heart failure with reduced ejection fraction?

A

lisinopril
spironolactone
metoprolol
losartan

132
Q

therapy for heart failure

A

ACEi or ARB
beta blocker (carvedilol, metoprolol)
mineralocorticoid receptor antagonist (spironolactone)

133
Q

58yo M in WV. L knee pain worsening x1wk. working on losing weight. jogs. pain worse when running, as day progresses, woken up from sleep. swelling, sore, generally unwell. pmhx HTN (amlodipine,HCTZ), CAD (stent, aspirin, metoprolol), psoriasis. pain w passive ROM throughout knee.

which of the following knee pathologies should be considered for your differential diagnosis

A

gout
osteoarthritis
pseudogout

134
Q

58yo M in WV. L knee pain worsening x1wk. working on losing weight. jogs. pain worse when running, as day progresses, woken up from sleep. swelling, sore, generally unwell. pmhx HTN (amlodipine,HCTZ), CAD (stent, aspirin, metoprolol), psoriasis. pain w passive ROM throughout knee.

what imaging or tests would you order based on your differential diagnosis?

A

fluid analysis
inflammatory markers
x-ray of the knee

135
Q

58yo M in WV. L knee pain worsening x1wk. working on losing weight. jogs. pain worse when running, as day progresses, woken up from sleep. swelling, sore, generally unwell. pmhx HTN (amlodipine,HCTZ), CAD (stent, aspirin, metoprolol), psoriasis. pain w passive ROM throughout knee.
xr: medial joint space narrowing, subchondral sclerosis, osteophytes. clear straw/yellow w no crystals. stopped running and walks instead. wears brace, ices. no pain, clicking, catching.

which of the following would be the next best steps in management?

A

continued home exercise
continued weight loss
ice and compression

136
Q

17yo M depression. diff conc, follwoing rule, always busy, constantly moving, disorganized. few months ago made appt for depression. now feels great, God-like, so smart, bought a motorcycle online, new sexual partners. God talks to him.

what conditions should be included in your differential?

A

bipolar disorder
brief psychotic disorder
methamphetamine use

137
Q

17yo M depression. diff conc, follwoing rule, always busy, constantly moving, disorganized. few months ago made appt for depression. now feels great, God-like, so smart, bought a motorcycle online, new sexual partners. God talks to him.
labs wnl. petition for psych placement and is admitted. dx bipolar I with active mania.

what treatments could be considered for the treatment of bipolar disorder?

A
aripriprazole
carbamazepine
lithium
risperidone
valproic acid
quetiapine

NOT CBT and NOT SSRIs

138
Q

17yo M depression. diff conc, follwoing rule, always busy, constantly moving, disorganized. few months ago made appt for depression. now feels great, God-like, so smart, bought a motorcycle online, new sexual partners. God talks to him.
labs wnl. petition for psych placement and is admitted. dx bipolar I with active mania.
started on lithium and quetiapine with goo d response. d/c from psych hospital.

you remember lithium has numerous side effects and special considerations including which of the following?

A
arrhythmias
cardiac malformations of fetus (in pregnant mother taking medication)
CNS depression
diminished renal concentrating ability
hypothyroidism
interaction with ibuprofen
139
Q

52yo M health fair anemia. labs were hgb 10.2, hct 33.4, leuk 14. colonoscopy last year w 1 polyp removed labs hgb 14.4, leuk 12.1, hct 44, mcv 88, TB 1.2, TChol 210, LDL 143. no pmhx. ROS +generalized fatigue, scleral icterus, nonpalpable rash on LE.

what is the name of this physical exam finding and what laboratory abnormality is it generally associated?

A

petechiae

assoc with thrombocytopenia

140
Q

52yo M health fair anemia. labs were hgb 10.2, hct 33.4, leuk 14. colonoscopy last year w 1 polyp removed labs hgb 14.4, leuk 12.1, hct 44, mcv 88, TB 1.2, TChol 210, LDL 143. no pmhx. ROS +generalized fatigue, scleral icterus, nonpalpable rash on LE.

given hx, labs, and PE findings, what workup should be obtained?

A

complete blood count with diferential
haptoglobin
liver profile including direct and indirect bilirubin
lactic acid dehydrogenase (LDH)

141
Q

52yo M health fair anemia. labs were hgb 10.2, hct 33.4, leuk 14. colonoscopy last year w 1 polyp removed labs hgb 14.4, leuk 12.1, hct 44, mcv 88, TB 1.2, TChol 210, LDL 143. no pmhx. ROS +generalized fatigue, scleral icterus, nonpalpable rash on LE.
wbc 24; hgb 9.8; plt 44; DBili 0.7; IBili 4.0; Alb 2.9; LDH 1490; Hapto < 30; smear -> abs inc in fx of leuks w/o dysplasia or blasts. normochromic normocyt anemia w reduce plts. abs reticulocytosis and spherocytes.

given above information, what further workup would you like to obtain?

A

direct antiglobulin test

142
Q

52yo M health fair anemia. labs were hgb 10.2, hct 33.4, leuk 14. colonoscopy last year w 1 polyp removed labs hgb 14.4, leuk 12.1, hct 44, mcv 88, TB 1.2, TChol 210, LDL 143. no pmhx. ROS +generalized fatigue, scleral icterus, nonpalpable rash on LE.
wbc 24; hgb 9.8; plt 44; DBili 0.7; IBili 4.0; Alb 2.9; LDH 1490; Hapto < 30; smear -> abs inc in fx of leuks w/o dysplasia or blasts. normochromic normocyt anemia w reduce plts. abs reticulocytosis and spherocytes.
DAT +. admitted and started on steroids. flow cyt -> monoclonal pop of mature B-cells. anemia worsens. shotty firm lymph nodes in axilla, groin, and neck < 5mm. repeat labs hgb 7.8, plt 12, wbc 36, LDH 240, retic 0.25%, hapto 150. peripheral smear -> no spherocytes, dec plt

what is the next step in management?

A

bone marrow biopsy

143
Q

52yo M health fair anemia. labs were hgb 10.2, hct 33.4, leuk 14. colonoscopy last year w 1 polyp removed labs hgb 14.4, leuk 12.1, hct 44, mcv 88, TB 1.2, TChol 210, LDL 143. no pmhx. ROS +generalized fatigue, scleral icterus, nonpalpable rash on LE.
wbc 24; hgb 9.8; plt 44; DBili 0.7; IBili 4.0; Alb 2.9; LDH 1490; Hapto < 30; smear -> abs inc in fx of leuks w/o dysplasia or blasts. normochromic normocyt anemia w reduce plts. abs reticulocytosis and spherocytes.
DAT +. admitted and started on steroids. flow cyt -> monoclonal pop of mature B-cells. anemia worsens. shotty firm lymph nodes in axilla, groin, and neck < 5mm. repeat labs hgb 7.8, plt 12, wbc 36, LDH 240, retic 0.25%, hapto 150. peripheral smear -> no spherocytes, dec plt
bone marrow -> malignant infiltration w dec erythrocyte and plt precursors. started on rituximab and chemo. looks unwell, lost 10lbs unint, night sweats, fatigue, fever, early satiety, febrile, diffuse enlarged lymph nodes, massively enlarged spleen.

this presentation is most concerning for what disease process?

A

Richter transformation

tx CLL -> sudden conversion of CLL to diffuse large B-cell lymphoma

144
Q

42yo F 3d continuous chest pain. sharp, substernal, radiates to neck. initially thought muscle train, but persisted. pmhx HTN (amlodipine), -card exam, -lung exam. EKG diffuse ST elevation

what additional imaging or labs would you order?

A

echocardiogram

trend cardiac enzymes

145
Q

42yo F 3d continuous chest pain. sharp, substernal, radiates to neck. initially thought muscle train, but persisted. pmhx HTN (amlodipine), -card exam, -lung exam. EKG diffuse ST elevation
echo: normal syst fxn, LVEF 65%, small pericardial effusion. cxr nl mediastinum. dx acute pericarditis. no travel, fevers night sweats, weight loss, arthritis, or famhx autoimmune

what additional testing or workup would you order at this time?

A

no additional testing

146
Q

rare etiologies of pericarditis

A

neoplasia - hx, wt loss, unexpl fevers, unexpl mass
TB - risk factors, endemic, fever, wt loss, nt sweats
purulent - severe sepsis, tamponade, risk of bacteremia/fungemia, HIV, risk of TB, any sev lung infxn
autoimmune - personal or famhx, assoc joint swelling, rash

147
Q

42yo F 3d continuous chest pain. sharp, substernal, radiates to neck. initially thought muscle train, but persisted. pmhx HTN (amlodipine), -card exam, -lung exam. EKG diffuse ST elevation
echo: normal syst fxn, LVEF 65%, small pericardial effusion. cxr nl mediastinum. dx acute pericarditis. no travel, fevers night sweats, weight loss, arthritis, or famhx autoimmune

what treatment would you offer at this time?

A

colchicine

ibuprofen

148
Q

88yo F in LT care, SOB. +palpitations, midsternal chest pain. dx adv dementia. hosp 3 mo ago for mechanical fall w femoral neck fx surgical repair. taken off ppx enoxaparin. pmhx alzheimers, htn (amlodipine and lisinopril), diastolic heart failure (furosemide), osteoporosis (alendronate). +tachyp. neg nucl stress test when c/o int dyspnea. no hx renal dysfxn.

which of the following diagnostic studies would you like to order for this patient?

A

complete blood count
CT angiography of the thorax
electrocardiogram
troponin

149
Q

88yo F in LT care, SOB. +palpitations, midsternal chest pain. dx adv dementia. hosp 3 mo ago for mechanical fall w femoral neck fx surgical repair. taken off ppx enoxaparin. pmhx alzheimers, htn (amlodipine and lisinopril), diastolic heart failure (furosemide), osteoporosis (alendronate). +tachyp. neg nucl stress test when c/o int dyspnea. no hx renal dysfxn.

which of the following medical diseases or treatments are known to be associated with increased risk for the suspected diagnosis?

A
congestive heart failure
inflammatory bowel disease
orthopedic surgery
malignancy
nephrotic syndrome
150
Q

88yo F in LT care, SOB. +palpitations, midsternal chest pain. dx adv dementia. hosp 3 mo ago for mechanical fall w femoral neck fx surgical repair. taken off ppx enoxaparin. pmhx alzheimers, htn (amlodipine and lisinopril), diastolic heart failure (furosemide), osteoporosis (alendronate). +tachyp. neg nucl stress test when c/o int dyspnea. no hx renal dysfxn.

which of the following abnormal ekg findings are associated with patient’s underlying diagnosis?

A

right bundle branch block
S1Q3T3
sinus tachycardia

151
Q

88yo F in LT care, SOB. +palpitations, midsternal chest pain. dx adv dementia. hosp 3 mo ago for mechanical fall w femoral neck fx surgical repair. taken off ppx enoxaparin. pmhx alzheimers, htn (amlodipine and lisinopril), diastolic heart failure (furosemide), osteoporosis (alendronate). +tachyp. neg nucl stress test when c/o int dyspnea. no hx renal dysfxn.
ekg new rbbb and sinus tach, trop and BNP mildy elev. abg -> resp alk w hypoxia and elev A-a gradient. CTA -> saddle PE. BP drops to 70/40. new JVD (8cm). start fluids & pressors, but blood pressure remains low with new-onset confusion.

how would you classify this patient’s pulmonary embolus?

A

massive pulmonary embolism

152
Q

88yo F in LT care, SOB. +palpitations, midsternal chest pain. dx adv dementia. hosp 3 mo ago for mechanical fall w femoral neck fx surgical repair. taken off ppx enoxaparin. pmhx alzheimers, htn (amlodipine and lisinopril), diastolic heart failure (furosemide), osteoporosis (alendronate). +tachyp. neg nucl stress test when c/o int dyspnea. no hx renal dysfxn.
ekg new rbbb and sinus tach, trop and BNP mildy elev. abg -> resp alk w hypoxia and elev A-a gradient. CTA -> saddle PE. BP drops to 70/40. new JVD (8cm). start fluids & pressors, but blood pressure remains low with new-onset confusion.
decision is made to proceed with tx given risk of death of PE.

which of the following should be administered immediately?

A

tissue plasminogen activator

153
Q

88yo F in LT care, SOB. +palpitations, midsternal chest pain. dx adv dementia. hosp 3 mo ago for mechanical fall w femoral neck fx surgical repair. taken off ppx enoxaparin. pmhx alzheimers, htn (amlodipine and lisinopril), diastolic heart failure (furosemide), osteoporosis (alendronate). +tachyp. neg nucl stress test when c/o int dyspnea. no hx renal dysfxn.
ekg new rbbb and sinus tach, trop and BNP mildy elev. abg -> resp alk w hypoxia and elev A-a gradient. CTA -> saddle PE. BP drops to 70/40. new JVD (8cm). start fluids & pressors, but blood pressure remains low with new-onset confusion.
decision is made to proceed with tx given risk of death of PE.
responds to therapy. O2 req decrease, becomes asymp. pt asks whether she needs to be on med for clots when she leaves.

what is the most appropriate response?

A

yes you should be on anticoagulation therapy for at least 3 months, but possibly longer. at a future time we can reassess the risks and benefits of anticoagulation.

154
Q

71yo M routine exam. since death of wife, unable to drive, got lost, can’t remember like he used to. moved to assistive living after water/electric shut off due to nonpayment. refuses group activities. used to enjoy painting. says hates painting. hx OA, stage IV CKD, CAD with quad bypass, father died of PD, mother of stroke. fine tremor with writing. walks slowly.

which of the following are the next best steps in management?

A

depression screening
detailed medication review
perform an objective cognitive assessment

155
Q

71yo M routine exam. since death of wife, unable to drive, got lost, can’t remember like he used to. moved to assistive living after water/electric shut off due to nonpayment. refuses group activities. used to enjoy painting. says hates painting. hx OA, stage IV CKD, CAD with quad bypass, father died of PD, mother of stroke. fine tremor with writing. walks slowly.
married for 40yrs before wife died of breast cancer.. forgetfulness, mild constipation, fatigue, and generalized malaise but otherwise ROS-.

what laboratory workup should be obtained in this patient?

A

thyroid stimulating hormone
B12
complete blood count

156
Q

71yo M routine exam. since death of wife, unable to drive, got lost, can’t remember like he used to. moved to assistive living after water/electric shut off due to nonpayment. refuses group activities. used to enjoy painting. says hates painting. hx OA, stage IV CKD, CAD with quad bypass, father died of PD, mother of stroke. fine tremor with writing. walks slowly.
married for 40yrs before wife died of breast cancer.. forgetfulness, mild constipation, fatigue, and generalized malaise but otherwise ROS-.
MoCA score 24.

which of the following is the most likely diagnosis?

A

pseudodementia

157
Q

71yo M routine exam. since death of wife, unable to drive, got lost, can’t remember like he used to. moved to assistive living after water/electric shut off due to nonpayment. refuses group activities. used to enjoy painting. says hates painting. hx OA, stage IV CKD, CAD with quad bypass, father died of PD, mother of stroke. fine tremor with writing. walks slowly.
married for 40yrs before wife died of breast cancer.. forgetfulness, mild constipation, fatigue, and generalized malaise but otherwise ROS-.
MoCA score 24.
started on sertraline. lab unremarkable. mri wnl. phq9 improved. still some diff with memory. forgets names, to take pills unless pillbox, 2/3 3 word recall.

what is the most likely diagnosis?

A

mild cognitive impairment

no impaired daily functioning

158
Q

42yo F c/o int epigastric and RUQ pain after eating, occasionally at night. within 1-2hrs after meal, dull. sometimes radiates to R shoulder/back. takes H2 blocker for GERD. no pmhx. BP 146/92. no melena/hematochezia. no tenderness, -Murphy’s.

what initial outpatient labs and imaging studies would you order?

A

liver function tests

right upper quadrant ultrasound

159
Q

42yo F c/o int epigastric and RUQ pain after eating, occasionally at night. within 1-2hrs after meal, dull. sometimes radiates to R shoulder/back. takes H2 blocker for GERD. no pmhx. BP 146/92. no melena/hematochezia. no tenderness, -Murphy’s.
labs wnl. u/s distended stone-filled gallbladder w wall thick 2mm, no pericholecystic fluid, normal CBD. has to wait 2mo for surg. occ chest tight worse w spicy foods. had some while watching tv. ok exercise. no smoke.

which of the following tests would be indicated?

A

EKG

160
Q

42yo F c/o int epigastric and RUQ pain after eating, occasionally at night. within 1-2hrs after meal, dull. sometimes radiates to R shoulder/back. takes H2 blocker for GERD. no pmhx. BP 146/92. no melena/hematochezia. no tenderness, -Murphy’s.
labs wnl. u/s distended stone-filled gallbladder w wall thick 2mm, no pericholecystic fluid, normal CBD. has to wait 2mo for surg. occ chest tight worse w spicy foods. had some while watching tv. ok exercise. no smoke.
ekg wnl. ED s/t worsening RUQ pain, vom, jaundice. confused. temp 103. tachy, 88/55. abd nondistended, +TTP RUQ, +asterixis.

based on this presentation in the ED, which of the following could be a differential diagnosis in this case?

A

acute viral hepatitis
ascending cholangitis
choledocholithiasis

161
Q

42yo F c/o int epigastric and RUQ pain after eating, occasionally at night. within 1-2hrs after meal, dull. sometimes radiates to R shoulder/back. takes H2 blocker for GERD. no pmhx. BP 146/92. no melena/hematochezia. no tenderness, -Murphy’s.
labs wnl. u/s distended stone-filled gallbladder w wall thick 2mm, no pericholecystic fluid, normal CBD. has to wait 2mo for surg. occ chest tight worse w spicy foods. had some while watching tv. ok exercise. no smoke.
ekg wnl. ED s/t worsening RUQ pain, vom, jaundice. confused. temp 103. tachy, 88/55. abd nondistended, +TTP RUQ, +asterixis.

select most appropriate tests for ED presentation.

A

blood cultures
chest x-ray
lactic acid
liver enzymes

162
Q

44yoM ED x2d gen wk, abd pain, diff urinating. diff initiating stream. no hx prostate. chronic back pain after constr accident, worsening over 2wks (ibuprofen). nightly alcohol, +tobacco, +heroin. +full suprapubic w pain. R/L patellar reflex 0, R biceps 1+. upgoing toes. mild arm weakness, dc grip strength, L foot clonus

given this patient’s presenting symptoms, what is/are the most important imaging study/studies to obtain?

A

MRI total spine with contrast

163
Q

44yoM ED x2d gen wk, abd pain, diff urinating. diff initiating stream. no hx prostate. chronic back pain after constr accident, worsening over 2wks (ibuprofen). nightly alcohol, +tobacco, +heroin. +full suprapubic w pain. R/L patellar reflex 0, R biceps 1+. upgoing toes. mild arm weakness, dc grip strength, L foot clonus
post-void residual 580cc. insert foley. temp 101.1, hr 104.

which antibiotics should be initiated empirically in this patient?

A

IV cefepime

IV vancomycin

164
Q

44yoM ED x2d gen wk, abd pain, diff urinating. diff initiating stream. no hx prostate. chronic back pain after constr accident, worsening over 2wks (ibuprofen). nightly alcohol, +tobacco, +heroin. +full suprapubic w pain. R/L patellar reflex 0, R biceps 1+. upgoing toes. mild arm weakness, dc grip strength, L foot clonus
post-void residual 580cc. insert foley. temp 101.1, hr 104.
started on abx w vanc + cefepime. MRI +abscess, osteomyelitis/discitis

what is the most important next step in management?

A

consult neurosurgery for decompressive surgery and abscess drainage

165
Q

44yoM ED x2d gen wk, abd pain, diff urinating. diff initiating stream. no hx prostate. chronic back pain after constr accident, worsening over 2wks (ibuprofen). nightly alcohol, +tobacco, +heroin. +full suprapubic w pain. R/L patellar reflex 0, R biceps 1+. upgoing toes. mild arm weakness, dc grip strength, L foot clonus
post-void residual 580cc. insert foley. temp 101.1, hr 104.
started on abx w vanc + cefepime. MRI +abscess, osteomyelitis/discitis
surg decompression. blood clx +ORSA.

what are the next best steps in management?

A

obtain an echocardiogram

repeat blood cultures

166
Q

44yoM ED x2d gen wk, abd pain, diff urinating. diff initiating stream. no hx prostate. chronic back pain after constr accident, worsening over 2wks (ibuprofen). nightly alcohol, +tobacco, +heroin. +full suprapubic w pain. R/L patellar reflex 0, R biceps 1+. upgoing toes. mild arm weakness, dc grip strength, L foot clonus
post-void residual 580cc. insert foley. temp 101.1, hr 104.
started on abx w vanc + cefepime. MRI +abscess, osteomyelitis/discitis
surg decompression. blood clx +ORSA.
TEE +23mm vegetation on tricuspid valve w severe regurg. after 7d, blood clx still pos. dev progr resp failure req 4L o2. cxr b/l pulm infiltrates w cavitary lesions.

what is the most important step in management?

A

surgical valve replacement

167
Q

5yo M c/o ear pain and fatigue. less active and more fussy at daycare. tugging ear. uncertain if utd on vax. mom and dad divorced. T 100.7.

without yet performing an eye, ears, nose, and throat examination, what conditions are highest on your differential diagnosis?

A

acute otitis media
otitis externa
upper respiratory infection

168
Q

5yo M c/o ear pain and fatigue. less active and more fussy at daycare. tugging ear. uncertain if utd on vax. mom and dad divorced. T 100.7.
red bulging TM w no motility. conservative tx. mom unlikely to f/u if sx do not improve. prescribe an abx. no allergies.

what antibiotic is most appropriate?

A

amoxicillin

169
Q

5yo M c/o ear pain and fatigue. less active and more fussy at daycare. tugging ear. uncertain if utd on vax. mom and dad divorced. T 100.7.
red bulging TM w no motility. conservative tx. mom unlikely to f/u if sx do not improve. prescribe an abx. no allergies.
manage w OMT using Galbreath technique.

which secondary bone is affected by this technique when draining the eustachain tube?

A

temporal bone

170
Q

28yoM. x5d runny nose, cong, sinus pressure +cough. no fever. bro similar sx a month ago. every year gets it, prescribes 2x abx in spring and fall, sometimes 3x. suggests zpack. no pmhx. married.

given this patient’s initial complaints and current level of information, which of the following should be considered on the differential diagnosis?

A
acute rhinocinusitis
bacterial rhinosinusitis
complicated rhinosinusitis
uncomplicated rhinosinusitis
viral rhinosinusitis
171
Q

28yoM. x5d runny nose, cong, sinus pressure +cough. no fever. bro similar sx a month ago. every year gets it, prescribes 2x abx in spring and fall, sometimes 3x. suggests zpack. no pmhx. married.
mild frontal HA, tender sinuses b/l. purulent d/c, edematous nasal turbinates.

which of the following are the next best steps in the management of this patient?

A

acetaminophen or ibuprofen

fluticasone nasal spray

172
Q

28yoM. x5d runny nose, cong, sinus pressure +cough. no fever. bro similar sx a month ago. every year gets it, prescribes 2x abx in spring and fall, sometimes 3x. suggests zpack. no pmhx. married.
mild frontal HA, tender sinuses b/l. purulent d/c, edematous nasal turbinates.
initially improves over the past 2d with worsening facial pain,, purulent d/c, fever. maxillary tooth pain.

what is the appropriate treatment for this patient’s condition?

A

oral amoxicillin/clavulanic acid

173
Q

28yoM. x5d runny nose, cong, sinus pressure +cough. no fever. bro similar sx a month ago. every year gets it, prescribes 2x abx in spring and fall, sometimes 3x. suggests zpack. no pmhx. married.
mild frontal HA, tender sinuses b/l. purulent d/c, edematous nasal turbinates.
initially improves over the past 2d with worsening facial pain,, purulent d/c, fever. maxillary tooth pain.
within 24hrs, worse facial pain and R eye swelling. hurts to look to sides, dec vision, diplopia. +fever. advise to go to ED. +proptosis, reduced EOM CN VI palsy.

what are the next best steps?

A

CT head and face with contrast
IV ceftriaxone OR IV ampicillin/sulbactam
IV vancomycin
ophthalmology consultation
IV metronidazole OR IV ampicillin/sulbactam

174
Q

25yo M MVA. restrained driver. unresponsive on scene, supraglottic airway. withdraws to pain, no open eyes. no limb deformities. diminished breath sounds b/l

what is the most appropriate next step?

A

establish definitive airway

175
Q

25yo M MVA. restrained driver. unresponsive on scene, supraglottic airway. withdraws to pain, no open eyes. no limb deformities. pulses 2+. est IV access. 2L crystalloids given w blood products.

what imaging studies should be obtained as adjuncts to the primary survey?

A

AP chest radiograph

FAST exam

176
Q

25yo M MVA. restrained driver. unresponsive on scene, supraglottic airway. withdraws to pain, no open eyes. no limb deformities. pulses 2+. est IV access. 2L crystalloids given w blood products.
FAST +fluid in pelvis. AP chest wnl. AP pelvis +open book fx. 3u pRBCs. CT head/cervical spine once stabilized.

what are the immediate next steps in management of patient’s pelvic fx?

A

apply pelvic binder
consult interventional radiology
laparotomy

177
Q

18yo F. gen abd pain, woke from sleep. +vomiting. worsening. ROS +anorexia. nl 2d ago. menses in 2wks. sexually active with new partner. on OCPs, but occasionally misses doses. T 100.4

what exam signs or findings would be most pertinent to look for?

A

cervical motion tenderness
Murphy sign
rebound tenderness with palpation

178
Q

18yo F. gen abd pain, woke from sleep. +vomiting. worsening. ROS +anorexia. nl 2d ago. menses in 2wks. sexually active with new partner. on OCPs, but occasionally misses doses. T 100.4
TTP worse near umbilicus & RLQ. no CVA. TTP in R adnexa, physiologic vaginal discharge.

what laboratory workup would you like to obtain on this patient?

A
basic metabolic panel
complete blood count
lactic acid
pregnancy test
urinalysis
179
Q

18yo F. gen abd pain, woke from sleep. +vomiting. worsening. ROS +anorexia. nl 2d ago. menses in 2wks. sexually active with new partner. on OCPs, but occasionally misses doses. T 100.4
TTP worse near umbilicus & RLQ. no CVA. TTP in R adnexa, physiologic vaginal discharge.
labs wbc 16, 14% bands; plt 550k, bicarb 19, beta-hCG neg, lactate 3.8

based on above, what imaging would you request?

A

CT of abdomen and pelvis with IV contrast

180
Q

18yo F. gen abd pain, woke from sleep. +vomiting. worsening. ROS +anorexia. nl 2d ago. menses in 2wks. sexually active with new partner. on OCPs, but occasionally misses doses. T 100.4
TTP worse near umbilicus & RLQ. no CVA. TTP in R adnexa, physiologic vaginal discharge.
labs wbc 16, 14% bands; plt 550k, bicarb 19, beta-hCG neg, lactate 3.8
CT A&P -> acute appy w/o perf. consult surg. T 102.3, HR 132, RR 22. severe abd pain, worse RLQ.

what interventions would you order before the patient undergoes surgery?

A

good peripheral IV access
IV opioid pain medication
resuscitation with IV fluid boluses followed by maintenance

181
Q

41yoM w/o chest pain. non-radiating, substernal, burning, worse at night. wake from sleep.

which of the following additional questions in the patient’s history would most assist you in obtaining an accurate diagnosis?

A

alleviating and aggravating factors
previous cardiac history
smoking history
timing

182
Q

41yoM w/o chest pain. non-radiating, substernal, burning, worse at night. wake from sleep.
always at night, between 30min -4hrs. +nausea. 2x/wk. worse w coffee and spicy. eats 1hr or less before bed. divorced, not with activity. restarted smoking. father died of MI at 53. TTP epigastrium.

what are the next best steps in management of this patient?

A

electrocardiogram

183
Q

41yoM w/o chest pain. non-radiating, substernal, burning, worse at night. wake from sleep.
always at night, between 30min -4hrs. +nausea. 2x/wk. worse w coffee and spicy. eats 1hr or less before bed. divorced, not with activity. restarted smoking. father died of MI at 53. TTP epigastrium.
ekg NSR w/o ST changes. discuss tx options and lifestyle mods. interested in lifestyle alone.

which of the following would you recommend as most appropriate lifestyle modifications for this patient?

A

avoid caffeine and spicy foods
elevate the head of the bed
weight loss

184
Q

41yoM w/o chest pain. non-radiating, substernal, burning, worse at night. wake from sleep.
always at night, between 30min -4hrs. +nausea. 2x/wk. worse w coffee and spicy. eats 1hr or less before bed. divorced, not with activity. restarted smoking. father died of MI at 53. TTP epigastrium.
ekg NSR w/o ST changes. discuss tx options and lifestyle mods. interested in lifestyle alone.
returns w mild improvement. ready to add medication.

which of the following are the most appropriate recommendations for this patient?

A

oral omeprazole

185
Q

41yoM w/o chest pain. non-radiating, substernal, burning, worse at night. wake from sleep.
always at night, between 30min -4hrs. +nausea. 2x/wk. worse w coffee and spicy. eats 1hr or less before bed. divorced, not with activity. restarted smoking. father died of MI at 53. TTP epigastrium.
ekg NSR w/o ST changes. discuss tx options and lifestyle mods. interested in lifestyle alone.
returns w mild improvement. ready to add medication.
inquire about screening for vascular condition often affecting males with hx of smoking, but would not generally be screened until 65.

which specific medical condition are you thinking of?

A

abdominal aortic aneurysm.

186
Q

41yoM w/o chest pain. non-radiating, substernal, burning, worse at night. wake from sleep.
always at night, between 30min -4hrs. +nausea. 2x/wk. worse w coffee and spicy. eats 1hr or less before bed. divorced, not with activity. restarted smoking. father died of MI at 53. TTP epigastrium.
ekg NSR w/o ST changes. discuss tx options and lifestyle mods. interested in lifestyle alone.
returns w mild improvement. ready to add medication.
inquire about screening for vascular condition often affecting males with hx of smoking, but would not generally be screened until 65.

which imaging modality is recommended in males 65-75 in order to screen for this condition?

A

ultrasound

187
Q

76yoM ED progr SOB. x4mo DOE, abd fullness after meals. used to be active. +orthopnea, decr exercise tolerance. hx DM (metformin, insulin glargine), aspirin. +tobacco, quit 9yrs ago. SOB to get to bedside commode. +systolic murmur at apex. split S2, decr w inspiration. +JVD. abd fullness, +hepatojugular reflux, +peripheral edema. placed on 3L O2.

what additional testing should be obtained in ED?

A

chest x-ray
basic metabolic panel
complete blood count
brain natriuretic peptide

188
Q

76yoM ED progr SOB. x4mo DOE, abd fullness after meals. used to be active. +orthopnea, decr exercise tolerance. hx DM (metformin, insulin glargine), aspirin. +tobacco, quit 9yrs ago. SOB to get to bedside commode. +systolic murmur at apex. split S2, decr w inspiration. +JVD. abd fullness, +hepatojugular reflux, +peripheral edema. placed on 3L O2.
Na 129; Cr 1.8; BNP 9840. Bili 1.6. cxr b/l infiltrates with central venous congestion and mod b/l pleural effusions w enlarged heart border.

what are the next most appropriate steps in management?

A

echocardiogram

IV furosemide

189
Q

76yoM ED progr SOB. x4mo DOE, abd fullness after meals. used to be active. +orthopnea, decr exercise tolerance. hx DM (metformin, insulin glargine), aspirin. +tobacco, quit 9yrs ago. SOB to get to bedside commode. +systolic murmur at apex. split S2, decr w inspiration. +JVD. abd fullness, +hepatojugular reflux, +peripheral edema. placed on 3L O2.
Na 129; Cr 1.8; BNP 9840. Bili 1.6. cxr b/l infiltrates with central venous congestion and mod b/l pleural effusions w enlarged heart border.
removes 8L fluid x3d. EF 25%. cardiac cath, severe triple vessel dz w >90% stenosis of LAD, RCA, and L circumflex, no stents.

what additional medications should be added for this patient before discharge?

A
po carvedilol (or metoprolol)
po lisinopril (any ACEi or ARB)
po atorvastatin (or rosuvastatin)
190
Q

76yoM ED progr SOB. x4mo DOE, abd fullness after meals. used to be active. +orthopnea, decr exercise tolerance. hx DM (metformin, insulin glargine), aspirin. +tobacco, quit 9yrs ago. SOB to get to bedside commode. +systolic murmur at apex. split S2, decr w inspiration. +JVD. abd fullness, +hepatojugular reflux, +peripheral edema. placed on 3L O2.
Na 129; Cr 1.8; BNP 9840. Bili 1.6. cxr b/l infiltrates with central venous congestion and mod b/l pleural effusions w enlarged heart border.
removes 8L fluid x3d. EF 25%. cardiac cath, severe triple vessel dz w >90% stenosis of LAD, RCA, and L circumflex, no stents.
f/u with cards. still mild dyspnea with strenuous exertion. no side effects from meds.

which of the following would you recommend for this patient?

A

coronary artery bypass grafting (CABG)

191
Q

32yo F. severe L pelvic pain xhrs +n/v, +diaphoresis. home pregnancy + 2wks ago. LMP 6wks ago. +chlamydia recently. trace blood in vaginal vault +adnexal tenderness. LLQ tender

which tests would you like to order?

A

pelvic ultrasound with transvaginal imaging if needed

quantitative HCG

192
Q

32yo F. severe L pelvic pain xhrs +n/v, +diaphoresis. home pregnancy + 2wks ago. LMP 6wks ago. +chlamydia recently. trace blood in vaginal vault +adnexal tenderness. LLQ tender
TVUS -> no intrauterine gestation, L adnexal mass. beta-hCG 2000. +diaphoretic, pale, worsening pain. 74/36. 137/min, 28/min. PIV placed, IVF. blood type A+. FAST -> pelvic free fluid. serum lactate 5.1.

what are the next most important steps in management?

A

consult OB/GYN for emergent surgery

blood transfusion

193
Q

32yo F. severe L pelvic pain xhrs +n/v, +diaphoresis. home pregnancy + 2wks ago. LMP 6wks ago. +chlamydia recently. trace blood in vaginal vault +adnexal tenderness. LLQ tender
TVUS -> no intrauterine gestation, L adnexal mass. beta-hCG 2000. +diaphoretic, pale, worsening pain. 74/36. 137/min, 28/min. PIV placed, IVF. blood type A+. FAST -> pelvic free fluid. serum lactate 5.1.

which of the following factors have the highest increased risk for ectopic pregnancy?

A

history of tubal surgery
history of pelvic inflammatory disease
previous ectopic pregnancy

194
Q

33yoM. new lump in L groin. after working out, gets larger sometimes. admits to heavy lifting. sometimes feels like it pops. reducible lump in L groin no erythema.

what is the most likely diagnosis of this patient

A

left inguinal hernia

195
Q

33yoM. new lump in L groin. after working out, gets larger sometimes. admits to heavy lifting. sometimes feels like it pops. reducible lump in L groin no erythema.
dx with reducible inguinal hernia. 2wks later intermittent sharp pain at site x3d. relieves by changing position. no pain at site, now cant push back in. new redness on skin surface. no n/v. not sig tender, but erythematous. abd soft nontender. serum lactate wnl.

which of the following are the most appropriate initial steps in management?

A

attempt manual reduction

196
Q

33yoM new onset, nonpruritic full body rash assoc w low grade fever, joint pains, sore throat x5d. +watery diarrhea w/o blood xfew weeks, lost 5kg. rash 6d ago on chest little red dots -> neck/back,arms. no IVDA. +maculopapular rash, worse over upper thorax/shoulders, some extension into palms. no target lesions, some mucocutaneous ulceration, between 5-10mm through mouth/pharynx.

what study/studies, if any, will you order for this patient at this time?

A

basic metabolic panel
complete blood count with differential
HIV testing
rapid plasma reagin

197
Q

33yoM new onset, nonpruritic full body rash assoc w low grade fever, joint pains, sore throat x5d. +watery diarrhea w/o blood xfew weeks, lost 5kg. rash 6d ago on chest little red dots -> neck/back,arms. no IVDA. +maculopapular rash, worse over upper thorax/shoulders, some extension into palms. no target lesions, some mucocutaneous ulceration, between 5-10mm through mouth/pharynx.
denies IVDA, +marijuana and MDMA, polygamy w/o barriers. UTD on vax. wbc 1.35, plt 93k, heterophile+

what is the most likely and concerning diagnosis given this patient’s presentation and current laboratory workup?

A

acute HIV infection

198
Q

33yoM new onset, nonpruritic full body rash assoc w low grade fever, joint pains, sore throat x5d. +watery diarrhea w/o blood xfew weeks, lost 5kg. rash 6d ago on chest little red dots -> neck/back,arms. no IVDA. +maculopapular rash, worse over upper thorax/shoulders, some extension into palms. no target lesions, some mucocutaneous ulceration, between 5-10mm through mouth/pharynx.
denies IVDA, +marijuana and MDMA, polygamy w/o barriers. UTD on vax. wbc 1.35, plt 93k, heterophile+

what specific tests for HIV needed to be ordered given your concern?

A

HIV RNA levels OR HIV viral load

HIV antigen/antibody testing OR HIV antibody testing

199
Q

33yoM new onset, nonpruritic full body rash assoc w low grade fever, joint pains, sore throat x5d. +watery diarrhea w/o blood xfew weeks, lost 5kg. rash 6d ago on chest little red dots -> neck/back,arms. no IVDA. +maculopapular rash, worse over upper thorax/shoulders, some extension into palms. no target lesions, some mucocutaneous ulceration, between 5-10mm through mouth/pharynx.
denies IVDA, +marijuana and MDMA, polygamy w/o barriers. UTD on vax. wbc 1.35, plt 93k, heterophile+
HIV RNA 130,000 copies/mL, CD4 640. resolution of acute sx.

when should this patient be initiated on antiretroviral therapy?

A

immediately

200
Q

33yoM new onset, nonpruritic full body rash assoc w low grade fever, joint pains, sore throat x5d. +watery diarrhea w/o blood xfew weeks, lost 5kg. rash 6d ago on chest little red dots -> neck/back,arms. no IVDA. +maculopapular rash, worse over upper thorax/shoulders, some extension into palms. no target lesions, some mucocutaneous ulceration, between 5-10mm through mouth/pharynx.
denies IVDA, +marijuana and MDMA, polygamy w/o barriers. UTD on vax. wbc 1.35, plt 93k, heterophile+
HIV RNA 130,000 copies/mL, CD4 640. resolution of acute sx.
lost to follow-up. 6yrs later worse exert dysp, fever, cough, weight loss x5wks. tachyp. white plaques on tongue and roof of mouth that scrape easily. cxr diffuse b/l interstitial infiltrates.

in addition to general treatment for sepsis and suspected respiratory infection, which of the following additional tests is indicateD?

A

arterial blood gas
LDH
1-3-beta-D-glucan (pneumocystis)

201
Q

33yoM new onset, nonpruritic full body rash assoc w low grade fever, joint pains, sore throat x5d. +watery diarrhea w/o blood xfew weeks, lost 5kg. rash 6d ago on chest little red dots -> neck/back,arms. no IVDA. +maculopapular rash, worse over upper thorax/shoulders, some extension into palms. no target lesions, some mucocutaneous ulceration, between 5-10mm through mouth/pharynx.
denies IVDA, +marijuana and MDMA, polygamy w/o barriers. UTD on vax. wbc 1.35, plt 93k, heterophile+
HIV RNA 130,000 copies/mL, CD4 640. resolution of acute sx.
lost to follow-up. 6yrs later worse exert dysp, fever, cough, weight loss x5wks. tachyp. white plaques on tongue and roof of mouth that scrape easily. cxr diffuse b/l interstitial infiltrates.
improves with abx. cd4 at dc is 38.

for which of the following organisms or infections should this patient receive prophylaxis?

A

mycobacterium avium complex (MAC)
pneumocystis pneumonia
toxoplasma gondii

202
Q

66yoF abd pain. intermittent x2d. epigastric. +n, +diaphoresis. pmhx hypothyroid, htn,, gerd, dm. stroke. meds low dose aspirin, levothyroxine, lisinopril, metformin, glyburide. 176/86. uncomfortable, shifting. RRR. soft minimall tender in epigastrium. no guarding.

which tests would you order at this time?

A

electrocardiogram

203
Q

66yoF abd pain. intermittent x2d. epigastric. +n, +diaphoresis. pmhx hypothyroid, htn,, gerd, dm. stroke. meds low dose aspirin, levothyroxine, lisinopril, metformin, glyburide. 176/86. uncomfortable, shifting. RRR. soft minimall tender in epigastrium. no guarding.
ekg 2mm ST depr in inferior leads. pain improves with bp control and nitroglycerin drip. trops elevated. uncomplicated cardiac catheterization and drug-eluting stent placement in RCA

which medications would you start/continue?

A

aspirin
atorvastatin
clopidogrel
metoprolol

204
Q

71yoF hx CAD, afib, htn, copd c/o dizziness x2d. feels unsteady like she’s going to fall over. unsure if she feels like she will lose consciousness. happens all the time. bumping into walls at home, holding onto objects. meds: aspirin, atorvastatin, metoprolol, amlodipine, lisinopril, rivaroxaban, and albuterol.

what are the important categories into which the general complaint of “dizziness” can be defined to help with clinical decision-making and to guide further workup?

A

disequilibrium
non-specific dizziness
presyncope
vertigo

205
Q

71yoF hx CAD, afib, htn, copd c/o dizziness x2d. feels unsteady like she’s going to fall over. unsure if she feels like she will lose consciousness. happens all the time. bumping into walls at home, holding onto objects. meds: aspirin, atorvastatin, metoprolol, amlodipine, lisinopril, rivaroxaban, and albuterol.
bed, chair, walking. unsteady. room is tilting. sometimes feels like bein gpushed over. worse with turning head. +n/v. continuous.

you want to ask a few more questions to help delineate the cause of this patient’s vertigo, what are other important details to ask regarding her HPI?

A
hearing loss
any recent infections
any headaches
tinnitus
any focal neurological symptoms
206
Q

71yoF hx CAD, afib, htn, copd c/o dizziness x2d. feels unsteady like she’s going to fall over. unsure if she feels like she will lose consciousness. happens all the time. bumping into walls at home, holding onto objects. meds: aspirin, atorvastatin, metoprolol, amlodipine, lisinopril, rivaroxaban, and albuterol.
bed, chair, walking. unsteady. room is tilting. sometimes feels like bein gpushed over. worse with turning head. +n/v. continuous.
no FND, 168/74.

what other physical exam maneuver or maneuvers should be performed to help determine the cause of this patient’s vertigo?

A

Dix-Hallpike Maneuver
HEENT examination including visualizing the ear drums with dedicated ocular testing
Head-Impulse-Nystagmus-Test-of-Skew (HINTS)

207
Q

71yoF hx CAD, afib, htn, copd c/o dizziness x2d. feels unsteady like she’s going to fall over. unsure if she feels like she will lose consciousness. happens all the time. bumping into walls at home, holding onto objects. meds: aspirin, atorvastatin, metoprolol, amlodipine, lisinopril, rivaroxaban, and albuterol.
bed, chair, walking. unsteady. room is tilting. sometimes feels like bein gpushed over. worse with turning head. +n/v. continuous.
no FND, 168/74.
Dix-Hallpike: immediate nystagmus w no delay, L beating, torsional, lasting 2mins. after sitting up, nystagmus is L beating on nystagmus testing. head impulse test wnl. test of skew and R eye takes 1 second to realign after uncovering. remainder of FND wnl.

what are the next best steps in management?

A

MRI brain
hospital admission
CT head without contrast

208
Q

63yo F c/o R hip pain for 3wks which has been worsening. dull, radiates to the groin to the knee. worse w weight bearing. bothers at night and sitting in chair. lost 10lbs x6mo. former smoker. TTP femur below greater trochanter. xray shows multiple lytic lesions of proximal femur and pelvis

which of the following questions are most important for determining etiology of patient’s lesions?

A

have you ever had a mammogram?
do you have any family history of cancer?
do you have a history of cancer?

209
Q

63yo F c/o R hip pain for 3wks which has been worsening. dull, radiates to the groin to the knee. worse w weight bearing. bothers at night and sitting in chair. lost 10lbs x6mo. former smoker. TTP femur below greater trochanter. xray shows multiple lytic lesions of proximal femur and pelvis.
mother died of breast cancer at 76. colonoscopy @61 wnl. mammogram 6y ago wnl, no follow up. UTD cervical screen.

what are the body areas/systems that you would like to focus on during your physical exam to help you determine etiology of suspected underlying malignancy?

A
breast exam
pulmonary exam
thyroid exam
abdominal exam
hip exam
neurological exam
cardiac exam
lymphatic exam
skin exam

at least 6 of the above systems

210
Q

63yo F c/o R hip pain for 3wks which has been worsening. dull, radiates to the groin to the knee. worse w weight bearing. bothers at night and sitting in chair. lost 10lbs x6mo. former smoker. TTP femur below greater trochanter. xray shows multiple lytic lesions of proximal femur and pelvis.
mother died of breast cancer at 76. colonoscopy @61 wnl. mammogram 6y ago wnl, no follow up. UTD cervical screen.
no lymphadenopathy. no thyroid/breast pathology.

what further imaging or labs would be most important initial steps to help determine source of patient’s malignancy?

A

complete metabolic panel
CT chest, abdomen, pelvis with positron emission tomography (PET) if available
serum and urine immunoelectrophoresis (SPEP, UPEP)

211
Q

26yoF c/o HA x48hrs. similar headaches for 2yrs, 1-2x/month. sometimes 7/10, throbbing, right side. n/v. photophobia. +smoke +obese. mom hx migraines, dad hx colon cancer. no FND.

what are the next best steps in the workup of this patient?

A

none
complete blood count
basic metabolic panel

212
Q

26yoF c/o HA x48hrs. similar headaches for 2yrs, 1-2x/month. sometimes 7/10, throbbing, right side. n/v. photophobia. +smoke +obese. mom hx migraines, dad hx colon cancer. no FND.

which of the following medications is appropriate to help manage her headaches at home?

A

acetaminophen orally
ibuprofen orally
sumatriptan orally

213
Q

26yoF c/o HA x48hrs. similar headaches for 2yrs, 1-2x/month. sometimes 7/10, throbbing, right side. n/v. photophobia. +smoke +obese. mom hx migraines, dad hx colon cancer. no FND.
does well w sumatriptan/naproxen x2y. getting worse and more frequent. MRI neg. labs wnl.

what is the next best step in the management of this patient?

A
start oral propranolol
start subcutaneous erenumab
cranial OMM
start oral amitriptyline
lifestyle modification
start oral verapamil
start oral venlafaxine
start oral valproate
start oral topiramate

*start a preventative tx

214
Q

84yoF found on bathroom floor with AMS and inability to stand. mom lives alone, but daughter checks on her every other day. pmhx htn (furosemide), afib (metoprolol + apixaban), “prediabetes.” 1step commands, symmetric pupils, confused. shortened L leg and bruising of L hip w palpable hematoma. no FND. in cervical collar. gluc 87.

what additional immediate management and lab workup should be included?

A

bolus of IV normal saline
creatine kinase
CT scan of head without contrast once stable
CT cervical spine without contrast once stable
EKG

215
Q

84yoF found on bathroom floor with AMS and inability to stand. mom lives alone, but daughter checks on her every other day. pmhx htn (furosemide), afib (metoprolol + apixaban), “prediabetes.” 1step commands, symmetric pupils, confused. shortened L leg and bruising of L hip w palpable hematoma. no FND. in cervical collar. gluc 87.
feels foggy. stuck on L side for a day. no head/neck pain. mild dysuria for a few days. dec po intake, mild nausea, generalized fatigue. Na 128, K 5.4, CK 17k. u/a hyaline casts + elev wbc.

what are the most appropriate next steps in management of this patient?

A

admission to the hospital
ceftriaxone IV
continued fluid administration
xray of the L hip

216
Q

84yoF found on bathroom floor with AMS and inability to stand. mom lives alone, but daughter checks on her every other day. pmhx htn (furosemide), afib (metoprolol + apixaban), “prediabetes.” 1step commands, symmetric pupils, confused. shortened L leg and bruising of L hip w palpable hematoma. no FND. in cervical collar. gluc 87.
feels foggy. stuck on L side for a day. no head/neck pain. mild dysuria for a few days. dec po intake, mild nausea, generalized fatigue. Na 128, K 5.4, CK 17k. u/a hyaline casts + elev wbc.
femoral neck fx. defer restarting apixaban. f/u 4wks later in clinic.

what are the next best steps in the management of this patient?

A

bisphosphonate administration

vitamin D level

217
Q

3yoM +SOB, +runny nose for a few days. vomited. diapers more wet, urinating into clothes. slightly premature, no sick contacts. dry mucus membranes. mild abd tenderness, tachypnea.

which tests would you like to order as part of initial workup?

A

basic metabolic panel
complete blood count
urinalysis

218
Q

3yoM +SOB, +runny nose for a few days. vomited. diapers more wet, urinating into clothes. slightly premature, no sick contacts. dry mucus membranes. mild abd tenderness, tachypnea.
gluc 450. Na 128, bicarb 8, Cr 1.4. AG 28. u/a +gluc, +ketones

what are the next best steps in management?

A

admission to the hospital under pediatrics service
intravenous fluid bolus
intravenous insulin

219
Q

21yoM rash. pruritic on elbows/forearms began a few weeks ago. 2nd time in last year. no sick contacts, no pmhx. T97.6.

what tests, if any, will you order for this patient at his time?

A

complete blood count
direct immunofluorescence microscopy
enzyme-linked immunosorbent assay for IgA tissue transglutaminase antibodies

220
Q

21yoM rash. pruritic on elbows/forearms began a few weeks ago. 2nd time in last year. no sick contacts, no pmhx. T97.6.
IF microscopy -> granular IgA deposits at the dermal papillae. ELISA for IgA TTA +.

what additional diagnostic testing, if any, will you order at this time?

A

esophagogastroduodenoscopy

small bowel biopsy

221
Q

21yoM rash. pruritic on elbows/forearms began a few weeks ago. 2nd time in last year. no sick contacts, no pmhx. T97.6.
IF microscopy -> granular IgA deposits at the dermal papillae. ELISA for IgA TTA +.
EGD -> scalloped duodenal folds. biopsy revealed absent villi and presence of intraepithelial lymphocytes.

how will you manage this patient at this time?

A

gluten-free diet

dapsone

222
Q

12yoF wcc. last seen 2y ago. missed 11yo wcc.

what screening evaluations, if any, will you perform at this time?

A

screening for depression

screening for tobacco, alcohol, and drug use (CRAFFT or HEADSS screening)

223
Q

12yoF wcc. last seen 2y ago. missed 11yo wcc.
tobacco, alcohol, drug neg. usually happy. wears seatbelt.

what immunizations, if any, will you order at this time?

A

meningococcal vaccine

human papillomavirus vaccine

224
Q

8mo M irritability x2d. not sleeping well, crying every time she tries to settle for a nap. pulling L ear. T102.5. bulging L TM opaque and erythematous. R opaque & mildly erythematous. no perfs.

what action(s), if any, would you like to take at this time?

A

amoxicillin treatment or amoxicillin course for 10d

ibuprofen or acetaminophen for pain control

225
Q

8mo M irritability x2d. not sleeping well, crying every time she tries to settle for a nap. pulling L ear. T102.5. bulging L TM opaque and erythematous. R opaque & mildly erythematous. no perfs.
completes amox. wcc 1mo later persisent left middle ear effusion. 6mo later fever, irrit, bulging L TM. 5th visit since initial. c/o language development. f/u 2wks later after amox/clav +mild conductive hearing loss

what actions will you take for this patient at this time?

A

refer to otolaryngologist for tympanostomy tubes

226
Q

8mo M irritability x2d. not sleeping well, crying every time she tries to settle for a nap. pulling L ear. T102.5. bulging L TM opaque and erythematous. R opaque & mildly erythematous. no perfs.
completes amox. wcc 1mo later persisent left middle ear effusion. 6mo later fever, irrit, bulging L TM. 5th visit since initial. c/o language development. f/u 2wks later after amox/clav +mild conductive hearing loss
referred to ENT for tubes. 3mo later foul-smelling drainage from L ear. afebrile. purulent.

what actions would you take at this time?

A

topical antibiotics with ofloxacin ear drops or ciprofloxacin-dexamethasone ear drops

227
Q

36yoF in ED diff breathing. SOB x24hr, mild chest pain w deep inspiration or coughing, no fever. +diaphoretic. +fine bibasilar rales.

what actions should be taken for this patient at this time?

A

D-dimer

228
Q

36yoF in ED diff breathing. SOB x24hr, mild chest pain w deep inspiration or coughing, no fever. +diaphoretic. +fine bibasilar rales.
ABG pH 7.45, pO2 90, pCO2 30. cxr: elev diaphragm. EKG -> ; D-dimer 1.4

what actions should be taken at this time?

A

CT pulmonary angiography

229
Q

36yoF in ED diff breathing. SOB x24hr, mild chest pain w deep inspiration or coughing, no fever. +diaphoretic. +fine bibasilar rales.
ABG pH 7.45, pO2 90, pCO2 30. cxr: elev diaphragm. EKG -> ; D-dimer 1.4
CT pulm angio -> filling defects in R interlobar pulmonary arteries consistent with pulmonary emboli.

what action, if any, should be taken at this time?

A

treatment with enoxaparin

230
Q

57yoM c/o intermittent chest pain. several weeks of tightness during daily evening walk. stops to rest and resolves within 5mins. when severe, radiates to L shoulder and jaw. pmhx untx HLD and smoking.

what tests, if any, should be ordered for this patient at this time?

A

electrocardiogram

exercise electrocardiogram

231
Q

57yoM c/o intermittent chest pain. several weeks of tightness during daily evening walk. stops to rest and resolves within 5mins. when severe, radiates to L shoulder and jaw. pmhx untx HLD and smoking.
trop 0.005. exercise ekg 2mm depressions of ST in V4, V5, V6 during treadmill. which correlate with pain. BP stable, EF 55%.

the most appropriate initial steps in management are:

A
beta blocker
nitroglycerin as needed for pain
aspirin
smoking cessation
statin
232
Q

57yoM c/o intermittent chest pain. several weeks of tightness during daily evening walk. stops to rest and resolves within 5mins. when severe, radiates to L shoulder and jaw. pmhx untx HLD and smoking.
trop 0.005. exercise ekg 2mm depressions of ST in V4, V5, V6 during treadmill. which correlate with pain. BP stable, EF 55%.
sx resolve after tx. 3yrs later, in ED for acute chest pain. at rest and exertion, relieved by sublingual nitroglycerin. squeezing, radiating x1hr. +diaphoretic, distressed. ekg no changes.

the most appropriate next steps in management are:

A
aspirin
atenolol
nitroglycerin
percutaneous coronary intervention
heparin
233
Q

81yoF hosp for CAP x12d. initially tx ceftriaxone and azithromycin. deter over 3d, switched to vanc and pip/taz. pmhx t2dm and htn. insulin, acetaminophen/ibuprofen. held anti-htn. preparing for d/c. temp 101 for first time in 6d. no change in exam. u/a microscopic hematuria and wbcs incl eosinophils

what actions if any will you take for this patient at this time?

A

discontinue piperacillin-tazobactam

discontinue ibuprofen/NSAIDs

234
Q

81yoF hosp for CAP x12d. initially tx ceftriaxone and azithromycin. deter over 3d, switched to vanc and pip/taz. pmhx t2dm and htn. insulin, acetaminophen/ibuprofen. held anti-htn. preparing for d/c. temp 101 for first time in 6d. no change in exam. u/a microscopic hematuria and wbcs incl eosinophils.
renal u/s: echogenic, normal sized kidneys and no obstructions. all nsaids and abx d/c. fever resolves but Cr remains elevated

how will you manage this patient at this time?

A

renal biopsy

235
Q

81yoF hosp for CAP x12d. initially tx ceftriaxone and azithromycin. deter over 3d, switched to vanc and pip/taz. pmhx t2dm and htn. insulin, acetaminophen/ibuprofen. held anti-htn. preparing for d/c. temp 101 for first time in 6d. no change in exam. u/a microscopic hematuria and wbcs incl eosinophils.
renal u/s: echogenic, normal sized kidneys and no obstructions. all nsaids and abx d/c. fever resolves but Cr remains elevated.
renal biopsy: interstitial infiltrate w lymphocytes and eosinophils and localized sites of interstitial fibrosis. histo -> acute interstitial nephritis. 6wks prednisone given. est GFR 25.

which actions, if any, will you take for this patient at this time?

A

discuss patient’s goals and preferences for renal replacement options
education of patient and family regarding renal replacement therapy

236
Q

36yo G2P2 w dysuria. burning x5d, urgency, inc frequency. new sex partner. no other pmhx. daily ocp. abd exam -> mild tenderness to palpation above the pubis. no CVA tenderness.

what tests, if any, will you order for this patient at this time?

A

pregnancy testing
urine dipstick for urinalysis
chlamydia trachomatis screen
neisseria gonorrhea screen

237
Q

36yo G2P2 w dysuria. burning x5d, urgency, inc frequency. new sex partner. no other pmhx. daily ocp. abd exam -> mild tenderness to palpation above the pubis. no CVA tenderness.
u/a nitrites and leuks positive, trace prot. full resolution. 3wks later, return for recurrent sx x6d. dysuria, frequency, and urgency. T101.4. abd exam mod TTP in suprapubic area

what additional testing, if any, will you order for this patient at this time?

A

urine culture with susceptibility
urinalysis
renal ultrasonography
repeat pregnancy test

238
Q

36yo G2P2 w dysuria. burning x5d, urgency, inc frequency. new sex partner. no other pmhx. daily ocp. abd exam -> mild tenderness to palpation above the pubis. no CVA tenderness.
u/a nitrites and leuks positive, trace prot. full resolution. 3wks later, return for recurrent sx x6d. dysuria, frequency, and urgency. T101.4. abd exam mod TTP in suprapubic area.
u/a nitrites, wbcs, prot, rbcs, beta-hCG pos. gram neg bacilli. renal u/s no anatomic abn. after antibiotics x3d, returns with n/v, fevers, chills, 102.4, stable. inc suprapubic tednerness and R CVA tenderness

how will you manage this patient?

A

hospital admission
IV antibiotics (ceftriaxone or cefepime)
prophylactic oral antibiotics to prevent recurrence - cephalexin or nitrofurantoin

239
Q

49yoF c/o lump in breast. famhx pos sister w breast cancer at 52. mammo 4mo ago. firm, immobile mass about 2-3mm in upper outer at 2oclock. no dimpling.

what tests if any will you order for this patient at this time

A

mammography
needle biopsy of the mass
ultrasonography of the breast

240
Q

49yoF c/o lump in breast. famhx pos sister w breast cancer at 52. mammo 4mo ago. firm, immobile mass about 2-3mm in upper outer at 2oclock. no dimpling.
mammo -> discrete density in upper outer of L breast with spiculated borders measuring 3mm. needle bx -> invasive ductal carcinoma

what additional testing if any will you order for this patient at this time?

A
HER2 overexpression
ER expression
PR expression
complete blood count
alanine aminotransferase
aspartate aminotransferase
alkaline phosphatase
sentinel lymph node biopsy
241
Q

49yoF c/o lump in breast. famhx pos sister w breast cancer at 52. mammo 4mo ago. firm, immobile mass about 2-3mm in upper outer at 2oclock. no dimpling.
mammo -> discrete density in upper outer of L breast with spiculated borders measuring 3mm. needle bx -> invasive ductal carcinoma.
ALT and AST elev. AlkPhos elev. HER2, ER, PR positive. sentinel lymph node -> nests and cords of cells with occasional glandular formation

what additional testing, if any, will you order for this patient at this time?

A

computed tomography scan of the abdomen

bone scan

242
Q

72yoM fatigue worsening x3mo. no pmhx. avid bicyclist, but fatigue impacting exercise tolerance. remote smoking >20yrs ago. sleeps well. no snore. HR 111. new systolic ejection murmur, +pallor.

what tests, if any, will you order for this patient at this time?

A

basic metabolic panel
complete blood count
hemoccult stool evaluation
thyroid stimulating hormone

243
Q

72yoM fatigue worsening x3mo. no pmhx. avid bicyclist, but fatigue impacting exercise tolerance. remote smoking >20yrs ago. sleeps well. no snore. HR 111. new systolic ejection murmur, +pallor.
Hgb 9, MCV 76, TSH 3.5. hemoccult stool pending.

what additional testing, if any, will you order at this time?

A
peripheral blood smear
reticulocyte count
serum ferritin level
serum iron level
total iron binding capacity
244
Q

72yoM fatigue worsening x3mo. no pmhx. avid bicyclist, but fatigue impacting exercise tolerance. remote smoking >20yrs ago. sleeps well. no snore. HR 111. new systolic ejection murmur, +pallor.
Hgb 9, MCV 76, TSH 3.5. hemoccult stool pending.
retic 0.2. ferritin 9, iron 32, TIBC 490. peripheral smear -> microcytic hypochromic rbcs with central pallor. hemoccult stool positive.

what additional testing, if any, will you order at this time?

A

colonoscopy

endoscopy

245
Q

72yoM fatigue worsening x3mo. no pmhx. avid bicyclist, but fatigue impacting exercise tolerance. remote smoking >20yrs ago. sleeps well. no snore. HR 111. new systolic ejection murmur, +pallor.
Hgb 9, MCV 76, TSH 3.5. hemoccult stool pending.
retic 0.2. ferritin 9, iron 32, TIBC 490. peripheral smear -> microcytic hypochromic rbcs with central pallor. hemoccult stool positive.
colonoscopy -> 2cm ulcerating mass in R colon. bx -> colonic adenocarcinoma.

some risk factors associated with this disease are:

A
alcohol use
insulin resistance
obesity
red meat consumption
tobacco use
246
Q

72yoM fatigue worsening x3mo. no pmhx. avid bicyclist, but fatigue impacting exercise tolerance. remote smoking >20yrs ago. sleeps well. no snore. HR 111. new systolic ejection murmur, +pallor.
Hgb 9, MCV 76, TSH 3.5. hemoccult stool pending.
retic 0.2. ferritin 9, iron 32, TIBC 490. peripheral smear -> microcytic hypochromic rbcs with central pallor. hemoccult stool positive.
colonoscopy -> 2cm ulcerating mass in R colon. bx -> colonic adenocarcinoma.

most appropriate next steps in management are:

A

surgical resection
abdominal CT scan
serum CEA levels

247
Q

63yoM routine. c/o chronic L knee pain. was active in youth. deep, throbbing medial joint pain worse after active days. avoiding exercise and “feels old and worthless.” knee stiff in AM x15mins. interferes with sleep 3-4x/wk. partial relief ibuprofen. no hx PUD. PE -> crepitus of L knee w passive movement, dec F ROM, mild knee swelling, ESR 12. x-ray -> joint space narrowing + osteophytes.

what actions, if any, should be taken for this patient at this time?

A

counsel patient on lifestyle modifications (diet and exercise)
oral diclofenac
refer patient to dietician for weight loss counseling

248
Q

63yoM routine. c/o chronic L knee pain. was active in youth. deep, throbbing medial joint pain worse after active days. avoiding exercise and “feels old and worthless.” knee stiff in AM x15mins. interferes with sleep 3-4x/wk. partial relief ibuprofen. no hx PUD. PE -> crepitus of L knee w passive movement, dec F ROM, mild knee swelling, ESR 12. x-ray -> joint space narrowing + osteophytes.
encourage exercise. loses 20lbs, takes oral diclofenac. wakes up 2x/wk with nocturnal pain. daughter’s wedding in 3wks.

what actions should be taken at this time?

A

intraarticular glucocorticoid injection of the left knee

referral to physical therapy

249
Q

63yoM routine. c/o chronic L knee pain. was active in youth. deep, throbbing medial joint pain worse after active days. avoiding exercise and “feels old and worthless.” knee stiff in AM x15mins. interferes with sleep 3-4x/wk. partial relief ibuprofen. no hx PUD. PE -> crepitus of L knee w passive movement, dec F ROM, mild knee swelling, ESR 12. x-ray -> joint space narrowing + osteophytes.
encourage exercise. loses 20lbs, takes oral diclofenac. wakes up 2x/wk with nocturnal pain. daughter’s wedding in 3wks.
4yrs later returns, healthy otherwise. s/p 3x steroid injections with resolution for 4wks. obtained hydrocodone to help him sleep. increasingly despondent, inability to help with chores

what is the best option for this patient at this time?

A

referral to orthopedics for total knee arthroplasty

250
Q

63yoM routine. c/o chronic L knee pain. was active in youth. deep, throbbing medial joint pain worse after active days. avoiding exercise and “feels old and worthless.” knee stiff in AM x15mins. interferes with sleep 3-4x/wk. partial relief ibuprofen. no hx PUD. PE -> crepitus of L knee w passive movement, dec F ROM, mild knee swelling, ESR 12. x-ray -> joint space narrowing + osteophytes.
encourage exercise. loses 20lbs, takes oral diclofenac. wakes up 2x/wk with nocturnal pain. daughter’s wedding in 3wks.
4yrs later returns, healthy otherwise. s/p 3x steroid injections with resolution for 4wks. obtained hydrocodone to help him sleep. increasingly despondent, inability to help with chores
referred for TKA. preop eval clear, has surgery.

what are the essential postoperative orders for this patient?

A

knee movement exercise (physical therapy)
pain management
thromboembolism prophylaxis

251
Q

34yoF urgent care 7th visit in last year. ongoing pain of abd, arms, legs, back. daily persists most of day. previous testing negative. referred to PCP but didn’t keep appt. 4 jobs in 3 years. ROS +daily fatigue, dif concentrating, 30lb wt gain in 18mo. diff falling asleep. PE wnl. TSH 2mo ago 1.3

what additional diagnostic assessment, if any, will you do at this time?

A

screening for depression

252
Q

34yoF urgent care 7th visit in last year. ongoing pain of abd, arms, legs, back. daily persists most of day. previous testing negative. referred to PCP but didn’t keep appt. 4 jobs in 3 years. ROS +daily fatigue, dif concentrating, 30lb wt gain in 18mo. diff falling asleep. PE wnl. TSH 2mo ago 1.3.
feels hopeless most days and no interest in doing any activities. few friends or family support. tearful.

what focused questions if any will you ask her at this time?

A

thoughts of suicide or suicidal ideation; thoughts of death or killing herself are acceptable alternatives

253
Q

34yoF urgent care 7th visit in last year. ongoing pain of abd, arms, legs, back. daily persists most of day. previous testing negative. referred to PCP but didn’t keep appt. 4 jobs in 3 years. ROS +daily fatigue, dif concentrating, 30lb wt gain in 18mo. diff falling asleep. PE wnl. TSH 2mo ago 1.3.
feels hopeless most days and no interest in doing any activities. few friends or family support. tearful.
frequent thoughts of harming herself, possibility of own death daily. bought a gun. takes gun out and looks at it and considers whether she’s brave enough

how will you manage this patient at this time?

A

referral for immediate inpatient psychiatric treatment

254
Q

35yoF routine. feels well, no complaints. no pmhx. social drinking, unmarried, sexually active. recent began work as social work, specializes in adoption from other countries. famhx sig MI at 60, breast cancer at 63. last exam 3yrs ago. pap neg at that time. last tetanus was 3yrs ago, unsure childhood.

what tests, if any, are recommended for this patient at this time?

A
CAGE questionnaire
chlamydia screening
gonorrhea screening
human immunodeficiency virus screening
screening for intimate partner violence
syphilis screening
255
Q

35yoF routine. feels well, no complaints. no pmhx. social drinking, unmarried, sexually active. recent began work as social work, specializes in adoption from other countries. famhx sig MI at 60, breast cancer at 63. last exam 3yrs ago. pap neg at that time. last tetanus was 3yrs ago, unsure childhood.

immunizations to which, if any, of the following are recommended for this patient?

A

hepatitis B

influenza

256
Q

35yoF routine. feels well, no complaints. no pmhx. social drinking, unmarried, sexually active. recent began work as social work, specializes in adoption from other countries. famhx sig MI at 60, breast cancer at 63. last exam 3yrs ago. pap neg at that time. last tetanus was 3yrs ago, unsure childhood.
labs no evidence of STI, no intimate partner violence, no CAGE.

what is the appropriate interval at which the patient should return to the clinic for a full screening, papanicolau smear, and HPV evaluation?

A

2 years

257
Q

23yoF with diff breathing. dyspnea on exertion for last few weeks. last couple days, breathing worsening and now associated with chest wall pain and dry cough, especially at night. no pmhx. exam -> b/l expiratory wheezing with poor air movement

what tests if any will you order for this patient at this time?

A

chest radiograph

spirometry

258
Q

23yoF with diff breathing. dyspnea on exertion for last few weeks. last couple days, breathing worsening and now associated with chest wall pain and dry cough, especially at night. no pmhx. exam -> b/l expiratory wheezing with poor air movement
PFT -> reduced FEV1 and FEV1/FVC, reverses with bronchodilators. cxr neg. cbc elev eosinophils.. .after 2x albuterol, pt sx return in 4hrs.

the most appropriate next steps in management are

A

continued short acting beta agonists
systemic glucocorticoids
avoidance of triggers
asthma education

259
Q

23yoF with diff breathing. dyspnea on exertion for last few weeks. last couple days, breathing worsening and now associated with chest wall pain and dry cough, especially at night. no pmhx. exam -> b/l expiratory wheezing with poor air movement
PFT -> reduced FEV1 and FEV1/FVC, reverses with bronchodilators. cxr neg. cbc elev eosinophils.. .after 2x albuterol, pt sx return in 4hrs.
after 1wk therapy w systemic glucocorticoids and int shortacting beta agonists, pt rerturns much improved. scheduled 3mo for repeat assessment. dyspnea and cough 3x/wk, takes albuterol with relief. four nights in past month waking from sleep

the most appropriate management at this time is

A

continue short acting beta agonist (albuterol)
add a daily inhaled glucocorticoid (fluticasone)
OR
add an inhaled glucocorticoid plus a short acting beta agonist concomitantly as needed
OR
low dose ICS-formoterol as needed

260
Q

38yoF hx prediabetes, inc menstrual bleeding q2-6wks, sometiems heavy. prev regular. weight gain. dec general energy w worsening fatigue. more sedentary. easy bruising. adult acne. thick terminal hair growth. previous pap wnl.

what are the most important next steps in evaluation of this patient?

A
pregnancy test
TSH
prolactin level
FSH level
coagulation studies
androgen levels
hemoglobin A1c
transvaginal ultrasound
ferritin
261
Q

38yoF hx prediabetes, inc menstrual bleeding q2-6wks, sometiems heavy. prev regular. weight gain. dec general energy w worsening fatigue. more sedentary. easy bruising. adult acne. thick terminal hair growth. previous pap wnl.
returns 4wks later. mild anemia hgb 9.3, hgbA1c 8.9, ferritin 10. gluc 232, nl Cr. freq waxing, issues w body hair most of life, worse since gaining weight. less sex drive. no children planned.
on OCPs, iron, metformin. returns 6mo later, menstration normalized, sig body hair.

what are the next best steps in management of this patient?

A
start oral antiandrogens
recheck iron stores
recheck A1c
check renal function and electrolytes
complete blood count
262
Q

58yoF f/u w orthopedist 2wks after L TKA. pain/swelling initially improved, but x5d pain in L calf. +tenderness, erytheam, swelling of L calf, +pitting edema. neurovasc intact. HCTZ, levothyroxine, conj estrogen pill

what is the most likely diagnosis?

A

deep vein thrombosis

263
Q

58yoF f/u w orthopedist 2wks after L TKA. pain/swelling initially improved, but x5d pain in L calf. +tenderness, erytheam, swelling of L calf, +pitting edema. neurovasc intact. HCTZ, levothyroxine, conj estrogen pill.
venous duplex +DVT L ant tibial v, popliteal v, and superficial femoral v. no hx, no anticoagulants.
d/c on rivaroxaban. unable to fill med d/t insurance, presents w chest discomfort, SOB, small hemoptysis.

best test to confirm her diagnosis?

A

thoracic CT angiography OR CT pulmonary angiography

264
Q

26yoF new pt. pmhx asthma and eczema. pshx nasal polyp excision and wisdom teeth extraction. no meds except rescue inhaler, using more frequently last few months. nonsmoker. grad student. sexually active in long term monogomous relationship. copper IUD 4yrs ago. last physical exam was >3yrs ago.

what exam or exams should you offer here at today’s visit?

A

pap smear

pulmonary function testing

265
Q

26yoF new pt. pmhx asthma and eczema. pshx nasal polyp excision and wisdom teeth extraction. no meds except rescue inhaler, using more frequently last few months. nonsmoker. grad student. sexually active in long term monogomous relationship. copper IUD 4yrs ago. last physical exam was >3yrs ago.
order spirometry and perform a Pap smear. notified sample was adequate for eval and some atypical squamous cells of undetermined significants (ASC-US). reflex HPV was not ordered. notify pt. se asks what is the most appropriate next step.

you recommend which of the following?

A

repeat Pap smear in 1 year